PDA

نسخه کامل مشاهده نسخه کامل : اتاق ریاضیات(طرح سؤالات)



صفحه ها : 1 2 3 4 5 6 7 8 9 [10] 11 12 13 14 15 16 17 18 19 20

Mohammad Hosseyn
21-09-2009, 07:17
1 _ یک سوال میتدیانه دیگر !!
در صفحه ی محورهای مختصات مجموعه جواب ای که در نامعادله ی زیر صدق می کند چگونه است ؟
[ برای مشاهده لینک ، لطفا با نام کاربری خود وارد شوید یا ثبت نام کنید ]
1 ) نمایش نوار موازی محور طولها
2 ) نمایش محور موازی محور عرض ها
3 ) نمایش مستطیل
3 ) نمایش دو خط موازی

......................................

در مورد اولی: اول از دو به توان X فاکتور بگیر... دو به توان x همواره مثبته... میمونه x+1/2>0 که جواب میشه x>- 0.5

در مورد دومی: سمت چپ برابره با x+1 به توان 3 ... از طرفین x+1 به توان دو رو ساده کن... چون x+1 به توان 2 مثبت هست، نیازی نیست در علامت نامساوی تغییری داده بشه... بعد از ساده کردن جواب میشه گزینه ی 3 امین جان بابت پاسخت خیلی ممنون .
در مورد دومی منظورت همون X-1 بود دیگه !! ... من یه ساعت داشتم فکر می کردم این X+1 کجاست .
بازم ممنون .
................................
2 _ این سوال رو هم لطف کنید جواب بدید ممنون میشم .
مجموعه جواب این نامعادله کدام است ؟
[ برای مشاهده لینک ، لطفا با نام کاربری خود وارد شوید یا ثبت نام کنید ]
البته فکر کنم با به پایه 2 در اوردن عددها بشه کاری کرد . اما من یه خورده سر مسئله گیج شدم .
...............................
3 _ جواب نامعادله زیر رو هم لطف کنید ممنون میشم :
[ برای مشاهده لینک ، لطفا با نام کاربری خود وارد شوید یا ثبت نام کنید ]
...............................
4 _ حدود x در نامعادله ی زیر چگونه هست ؟
[ برای مشاهده لینک ، لطفا با نام کاربری خود وارد شوید یا ثبت نام کنید ]

................................
5 _ حاصل عبارت زیر :
[ برای مشاهده لینک ، لطفا با نام کاربری خود وارد شوید یا ثبت نام کنید ]







من هر چی حل می کنم به جواب -1-x میرسم !!! اما بین گزینه ها نیست .
..............................

amintnt
21-09-2009, 12:25
1 _ یک سوال میتدیانه دیگر !!
در صفحه ی محورهای مختصات مجموعه جواب ای که در نامعادله ی زیر صدق می کند چگونه است ؟
[ برای مشاهده لینک ، لطفا با نام کاربری خود وارد شوید یا ثبت نام کنید ]
1 ) نمایش نوار موازی محور طولها
2 ) نمایش محور موازی محور عرض ها
3 ) نمایش مستطیل
3 ) نمایش دو خط موازی

......................................
امین جان بابت پاسخت خیلی ممنون .
در مورد دومی منظورت همون X-1 بود دیگه !! ... من یه ساعت داشتم فکر می کردم این X+1 کجاست .
بازم ممنون .
................................
2 _ این سوال رو هم لطف کنید جواب بدید ممنون میشم .
مجموعه جواب این نامعادله کدام است ؟
[ برای مشاهده لینک ، لطفا با نام کاربری خود وارد شوید یا ثبت نام کنید ]
البته فکر کنم با به پایه 2 در اوردن عددها بشه کاری کرد . اما من یه خورده سر مسئله گیج شدم .
...............................
3 _ جواب نامعادله زیر رو هم لطف کنید ممنون میشم :
[ برای مشاهده لینک ، لطفا با نام کاربری خود وارد شوید یا ثبت نام کنید ]
...............................
4 _ حدود x در نامعادله ی زیر چگونه هست ؟
[ برای مشاهده لینک ، لطفا با نام کاربری خود وارد شوید یا ثبت نام کنید ]

................................
5 _ حاصل عبارت زیر :
[ برای مشاهده لینک ، لطفا با نام کاربری خود وارد شوید یا ثبت نام کنید ]





من هر چی حل می کنم به جواب -1-x میرسم !!! اما بین گزینه ها نیست .
..............................

در صفحه ی محورهای مختصات مجموعه جواب ای که در نامعادله ی زیر صدق می کند چگونه است ؟
[ برای مشاهده لینک ، لطفا با نام کاربری خود وارد شوید یا ثبت نام کنید ]
1 ) نمایش نوار موازی محور طولها
2 ) نمایش محور موازی محور عرض ها
3 ) نمایش مستطیل
3 ) نمایش دو خط موازیجواب نامعادله میشه: [ برای مشاهده لینک ، لطفا با نام کاربری خود وارد شوید یا ثبت نام کنید ]

مقدار تابع (Y) میتونه روی محور عرض ها نوسان کنه... بنابراین فاصله ی بین بازه های [ برای مشاهده لینک ، لطفا با نام کاربری خود وارد شوید یا ثبت نام کنید ] به موازات محور عرض ها جارو میشه... شاید بشه گفت گزینه ی 3... یعنی دو مستطیل به وجود میاد... ان شاءالله که همینه!


_ این سوال رو هم لطف کنید جواب بدید ممنون میشم .
مجموعه جواب این نامعادله کدام است ؟
[ برای مشاهده لینک ، لطفا با نام کاربری خود وارد شوید یا ثبت نام کنید ]


[ برای مشاهده لینک ، لطفا با نام کاربری خود وارد شوید یا ثبت نام کنید ]

[ برای مشاهده لینک ، لطفا با نام کاربری خود وارد شوید یا ثبت نام کنید ]

که همیشه برقراره... البته لازم به ذکره خیلی راحت میشد همون عبارت اولی رو تعیین علامت کرد.... مجموعه جواب همون R یا اعداد حقیقی میشه....


_ جواب نامعادله زیر رو هم لطف کنید ممنون میشم :
[ برای مشاهده لینک ، لطفا با نام کاربری خود وارد شوید یا ثبت نام کنید ]
...............................
4 _ حدود x در نامعادله ی زیر چگونه هست ؟
[ برای مشاهده لینک ، لطفا با نام کاربری خود وارد شوید یا ثبت نام کنید ]هر دو سوالی یکی هستن... متاسفانه چون با این سایت Latex خیلی کار نکردم از کار کردن باهاش عاجزم.... همینطوری توضیح میدم... طرف دوم رو میارین طرف اول... مخرج مشترک میگیرین... صورت میشه 3- و مخرج میشه [ برای مشاهده لینک ، لطفا با نام کاربری خود وارد شوید یا ثبت نام کنید ]
تعیین علامت میکنید... جواب میشه [ برای مشاهده لینک ، لطفا با نام کاربری خود وارد شوید یا ثبت نام کنید ]


5 _ حاصل عبارت زیر :
[ برای مشاهده لینک ، لطفا با نام کاربری خود وارد شوید یا ثبت نام کنید ]قدر مطلق اول در بازه ی مورد نظر همواره منفی هست.... و قدر مطلق دوم هم همواره مثبت... بنابراین:
[ برای مشاهده لینک ، لطفا با نام کاربری خود وارد شوید یا ثبت نام کنید ]

امیدوارم جایی اشتباه نکرده باشم... امتحان آیین نامه دارم و همچنین عجله!

matin56
22-09-2009, 20:23
دوستان عزیز: ایده ای دارید که از تابعی که متغیرش به صورت حد بالایی یک مجموع (سیگما) در آن ظاهر شده چطور میشه انتگرال گرفت؟ به زبان متمتیکا یعنی این شکلی مثلا:


[[f(x) = Sum[((((k - x))^k)/k!)*Exp[-1*(k - x)], {k, 0, floor[x

maryam.jolghazi
25-09-2009, 21:45
یه مقدارم راجع به سوالات دانشگاهی بحث کنید بد نیست
مرسسسسسی

ostadonline
26-09-2009, 18:21
سلام. شرمنده اما میشه در مورد اثبات این سیگما راهنمایی بفرمایید:

[ برای مشاهده لینک ، لطفا با نام کاربری خود وارد شوید یا ثبت نام کنید ] ([ برای مشاهده لینک ، لطفا با نام کاربری خود وارد شوید یا ثبت نام کنید ])

پیشاپیش ممنون...

k1kz
27-09-2009, 23:07
دوستان عزیز: ایده ای دارید که از تابعی که متغیرش به صورت حد بالایی یک مجموع (سیگما) در آن ظاهر شده چطور میشه انتگرال گرفت؟ به زبان متمتیکا یعنی این شکلی مثلا:


[[f(x) = Sum[((((k - x))^k)/k!)*Exp[-1*(k - x)], {k, 0, floor[x


سلام به همه دوستان
من به خاطر مسایل شغلی خیلی منظم نمیتونم بیام اینترنت.ولی به هر حال خوشحالم دوباره اومدم و به همه رفقای قدیمی هم سلام عرض میکنم.

صورت مساله شما: انتگرال گیری از :[ برای مشاهده لینک ، لطفا با نام کاربری خود وارد شوید یا ثبت نام کنید ]بر حسب x


این سوال شما به نظرم خیلی جالبه من چند راه حل به نظرم میرسه :

1) استفاده از سری تیلور عبارت داخل سیگما و بسط آن و اعمال مجموع روی بسط ها و بدست آوردن یک رابطه
صریح بر حسب x (که احتمالا به صورت یک سری چندگانه بر حسب x با ضرایب شامل توابع بازگشتی هستند)
اینطوری میتونید شروع کنید:
[ برای مشاهده لینک ، لطفا با نام کاربری خود وارد شوید یا ثبت نام کنید ]


2)استفاده از سری های مجانبی(Asymptotic) و دنبال کردن مسایل بالا به این صورت:

[ برای مشاهده لینک ، لطفا با نام کاربری خود وارد شوید یا ثبت نام کنید ]



3)استفاده از حاصلضرب سری های بسط عبارات داخل سیگما به صورت جداگانه و اعمال روشهای حاصلضرب سریها .به اینصورت :
[ برای مشاهده لینک ، لطفا با نام کاربری خود وارد شوید یا ثبت نام کنید ]

4)و یه راه متفاوت اینکه اصلا از تعریف انتگرال یعنی حد مجموع استفاده کنید و یا از روشهای عددی انتگرال گیری مثل روش گوش با نقاط معلوم استفاده کنید.

ولی اینها یا روشهای مشابه مستلزم زمان طولانی برای محاسبه هستند من احساس میکنم شما در جریان حل یک مساله دیگری به این عبارت برخوردید و راه حل مناسبی رو انتخاب نکردید که زمان کار رو خیلی پایین میاره ,درسته؟ میشه بیشتر توضیح بدین؟


>

k2l
28-09-2009, 19:30
سلام.ببخشید اگه این سوال جاش اینجا نیست،اما ممنون میشم کمکم کنید.

در چه صورت این عبارت یک عبارت اصم هست؟ [ برای مشاهده لینک ، لطفا با نام کاربری خود وارد شوید یا ثبت نام کنید ]
(طرف دوم به توان m روی n)

sepehr_x50
28-09-2009, 19:45
سلام.

دوستان این جمله یعنی چی؟

اعداد گویا و اصم (گنگ) در هم چگالند!

ممنون

CATALONIA
28-09-2009, 19:47
نمی دونستم چطوری سوالا مو مطرح کنم ولی به هر صورت
اگه میشه یه کم ساده این مسئله ها رو حل کنین من سوم راهنمایی ام ولی در حد دبیرستان بگینم طوری نیست
اگه میشه حتما تا سه شنبه شب جواب ها رو بگین چون برای چهار شنیه میخوامش
1-باقی مانده ی تقسیم 5 به توان 1374 بر 31 چقدر است؟
2-مجموع مربعات (مجذورات ) دو عدد اول 293 است . تفاضل آن دو عدد چقدر است ؟
3- اگر a و b دو عدد اول متوالی باشند و x یک عدد طبیعی به طوری که 2x - a = b باشد . عدد x اول است یا مرکب ؟
ممنون میشم که خیلی سریع شما پروفسور ها جواب منو بدین
ممنون

ali_hp
28-09-2009, 20:17
سلام.

دوستان این جمله یعنی چی؟

اعداد گویا و اصم (گنگ) در هم چگالند!

ممنون
سلام
احتمالا منظور اینه که بین هر دو عدد گویا عددی گنگ وجود دارد،و بین هر دو عدد گنگ عددی گویا وجود دارد.

ali_hp
28-09-2009, 20:32
[quote=CATALONIA;4265632]نمی دونستم چطوری سوالا مو مطرح کنم ولی به هر صورت
اگه میشه یه کم ساده این مسئله ها رو حل کنین من سوم راهنمایی ام ولی در حد دبیرستان بگینم طوری نیست
اگه میشه حتما تا سه شنبه شب جواب ها رو بگین چون برای چهار شنیه میخوامش
1-باقی مانده ی تقسیم 5 به توان 1374 بر 31 چقدر است؟
2-مجموع مربعات (مجذورات ) دو عدد اول 293 است . تفاضل آن دو عدد چقدر است ؟
3- اگر a و b دو عدد اول متوالی باشند و x یک عدد طبیعی به طوری که 2x - a = b باشد . عدد x اول است یا مرکب ؟
ممنون میشم که خیلی سریع شما پروفسور ها جواب منو بدین
ممنون[/quote
]سلام
1)راهنمایی:باقیمانده پنج به توان سه بر 31 برابر یک است!
2)اگر هردو عدد فرد باشند،مجموع مربعات آنها زوج می شود،پس یکی از اعداد باید 2 باشد!
3)مرکب است،چون x بین a و b قرار می گیرد،و a وb دو عدد اول متوالی هستند!

ali_hp
28-09-2009, 20:40
سلام.ببخشید اگه این سوال جاش اینجا نیست،اما ممنون میشم کمکم کنید.

در چه صورت این عبارت یک عبارت اصم هست؟ [ برای مشاهده لینک ، لطفا با نام کاربری خود وارد شوید یا ثبت نام کنید ]
(طرف دوم به توان m روی n)
سلام
در صورتی که m بر n بخش پذیر نباشد.

CATALONIA
28-09-2009, 21:49
[/quote
]سلام
1)راهنمایی:باقیمانده پنج به توان سه بر 31 برابر یک است!
2)اگر هردو عدد فرد باشند،مجموع مربعات آنها زوج می شود،پس یکی از اعداد باید 2 باشد!
3)مرکب است،چون x بین a و b قرار می گیرد،و a وb دو عدد اول متوالی هستند![/quote]


میشه یه کم 

نه بیشتر از یه کم 
توضیح بدین 
ما سوم راهنمایی هستیم 

مثله شما عزیزان که پروفسور نیستیم
اولی هم پنج به توانه 1374 

ali_hp
28-09-2009, 22:27
[/quote
]سلام
1)راهنمایی:باقیمانده پنج به توان سه بر 31 برابر یک است!
2)اگر هردو عدد فرد باشند،مجموع مربعات آنها زوج می شود،پس یکی از اعداد باید 2 باشد!
3)مرکب است،چون x بین a و b قرار می گیرد،و a وb دو عدد اول متوالی هستند!


میشه یه کم

نه بیشتر از یه کم
توضیح بدین
ما سوم راهنمایی هستیم

مثله شما عزیزان که پروفسور نیستیم
اولی هم پنج به توانه 1374 [/quote]
اگه بازم جاییش مبهم بود بگید که بیشتر توضیح بدم.
1)باقیمانده 5 به توان 3 بر 31 برابر 1 است.(حساب کنید!)از طرفی:
5 به توان 1374 =125 به توان 458
حال با توجه به اینکه باقیمانده حاصلضرب دو عدد برابر است با باقیمانده حاصلضرب باقیمانده های دو عدد،باقیمانده هر توانی از 125 بر 31 برابر یک می شود،پس جواب یک است!
2)یکی از این اعداد اول باید دو باشد،در غیر اینصورت هر دو عدد فرد هستند،و مجذورشان نیز فرد است،پس مجموع مجذورشان زوج است،و نمی تواند برابر 293 باشد،پس یکی از اعداد 2 است و دیگری جذر 293-4=289
یعنی 17،که تفاضل دو عدد میشه 15
3)a رو ببرید طرف دیگه معادله و طرفینو تقشیم بر دو کنید،میبینید که x میانگین دو عدد a و b است(مجموعشان تقسیم بر دو) پس x بین a و b قرار دارد،از طرفی چون a و b دو عدد اول متوالی هستند،هر عدد بین آنها مرکب است،پس x نیز مرکب است.

CATALONIA
29-09-2009, 14:30
  
اگه بازم جاییش مبهم بود بگید که بیشتر توضیح بدم.
1)باقیمانده 5 به توان 3 بر 31 برابر 1 است.(حساب کنید!)از طرفی:
5 به توان 1374 =125 به توان 458
حال با توجه به اینکه باقیمانده حاصلضرب دو عدد برابر است با باقیمانده حاصلضرب باقیمانده های دو عدد،باقیمانده هر توانی از 125 بر 31 برابر یک می شود،پس جواب یک است!
2)یکی از این اعداد اول باید دو باشد،در غیر اینصورت هر دو عدد فرد هستند،و مجذورشان نیز فرد است،پس مجموع مجذورشان زوج است،و نمی تواند برابر 293 باشد،پس یکی از اعداد 2 است و دیگری جذر 293-4=289
یعنی 17،که تفاضل دو عدد میشه 15
3)a رو ببرید طرف دیگه معادله و طرفینو تقشیم بر دو کنید،میبینید که x میانگین دو عدد a و b است(مجموعشان تقسیم بر دو) پس x بین a و b قرار دارد،از طرفی چون a و b دو عدد اول متوالی هستند،هر عدد بین آنها مرکب است،پس x نیز مرکب است.


ممنون 
اگه هر کدوم از اینها فرمولی هم داره لطفا بگین


انصافا اینا برای سوم راهنمایی سخت نیست

ali_hp
29-09-2009, 16:05
ممنون
اگه هر کدوم از اینها فرمولی هم داره لطفا بگین


انصافا اینا برای سوم راهنمایی سخت نیست
برای کسی که اولین بار باشه که این سبک سوالا رو می بینه،واقعا سخته!مثلا برای یک باید حداقل چند تا از ویژگیهای باقیمانده رو بدونین،و به مقدار کافی هم تمرین حل کرده باشین،اما هر سه سوال طوری هستند که اگر به مقدار کافی تمرین کنید،به سادگی میتونید مساله های مشابهشونو حل کنید....
برای سوال اول باید مهارت خودتونو در محاسبه باقیمانده افزایش بدین که می تونین از نکات زیر استفاده کنید:
باقیمانده حاصلضرب برابر است با باقیمانده حاصلضرب باقیمانده ها.
باقیمانده مجموع برابر است با باقیمانده مجموع باقیمانده ها.
با توجه به دو نکته بالا در محاسبه باقیمانده یک عبارت متشکل از ضرب و جمع،به جای هر عدد می توان باقیمانده آن را قرار داد.مثلا می خواهیم باقیمانده 2009*1388+145*155 را بر 5 حساب کنیم،یک روش محاسبه مقدار عبارت و بعد حساب کردن باقیمانده است.با قرار دادن باقیمانده اعداد بر 5 به جای خود اعداد به دست می آید:
0*0+3*4=12 و باقیمانده 12 نیز بر 5 می شود 2!
در مورد مساله دوم هم همیشه حواستون به زوجیت باشه!مثلا اینجا از اینکه مجموع مربع دو عدد اول فرد شده،نتیجه گرفتیم که یکیشون زوجه و مساله حل شد!
سوال سوم هم دقت و نکته سنجی می خواد...

sd70
01-10-2009, 22:44
سلام خسته نباشيد

جلسه اول کلاس رياضي يه مشکل برام پيش اومد نتونستم برم. البته نرفتنم به کلاس عمدي نبود ولي خب فکرش رو هم نميکردم اين استاده اين جلسه اول اين همه جزوه و مزخرفات بگه. امروز که جلسه دوم بود رفتم نشستم سر کلاس داشت مخم دود ميکرد هيچي نفهميدم از چيزايي که گفت. (چون جلسه قبلش رو نرفته بودم)

يه سري شر و ور راجب نميدونم cisθ و arg و نميدونم ريشه عدد مختلط و اين حرفا ميگفت.

مثال هايي که گفت:

حاصل اين توان (1 + راديکال 3 آي به توان 48) را بدست بياريد.

يا مثلا چهار ريشه عدد منفي يک رو بدست بياريد..

سه ريشه عدد 8i رو بدست بياريد و از اين جور مزخرفات.

البته تمام اين مثال ها رو خودش حل کرد ولي من چون جلسه قبلش تو کلاس نبودم هيچي نفهميدم.

ممنون ميشم يکي از دوستان در مورد اين مزخرفات توضيح بده تا جلسه قبلي که نرفته بودم جبران بشه و بتونم درسم رو بفهمم. ممنون

attractive_girl
02-10-2009, 10:34
سلام . لطفا اینو حل کنید .جوابش چی میشه؟

[ برای مشاهده لینک ، لطفا با نام کاربری خود وارد شوید یا ثبت نام کنید ]

emgjey
02-10-2009, 17:28
سلام خسته نباشيد

جلسه اول کلاس رياضي يه مشکل برام پيش اومد نتونستم برم. البته نرفتنم به کلاس عمدي نبود ولي خب فکرش رو هم نميکردم اين استاده اين جلسه اول اين همه جزوه و مزخرفات بگه. امروز که جلسه دوم بود رفتم نشستم سر کلاس داشت مخم دود ميکرد هيچي نفهميدم از چيزايي که گفت. (چون جلسه قبلش رو نرفته بودم)

يه سري شر و ور راجب نميدونم cisθ و arg و نميدونم ريشه عدد مختلط و اين حرفا ميگفت.

مثال هايي که گفت:

حاصل اين توان (1 + راديکال 3 آي به توان 48) را بدست بياريد.

يا مثلا چهار ريشه عدد منفي يک رو بدست بياريد..

سه ريشه عدد 8i رو بدست بياريد و از اين جور مزخرفات.

البته تمام اين مثال ها رو خودش حل کرد ولي من چون جلسه قبلش تو کلاس نبودم هيچي نفهميدم.

ممنون ميشم يکي از دوستان در مورد اين مزخرفات توضيح بده تا جلسه قبلي که نرفته بودم جبران بشه و بتونم درسم رو بفهمم. ممنون
به کتب مرجع رجوع کنید و مبحث اعداد مختلط رو مطالعه کنید

davy jones
03-10-2009, 15:40
سلام . لطفا اینو حل کنید .جوابش چی میشه؟

[ برای مشاهده لینک ، لطفا با نام کاربری خود وارد شوید یا ثبت نام کنید ]

i و u چی هستن؟

.:HAMED:.
03-10-2009, 22:17
i و u چی هستن؟
حتما متغیر ها هستن دیگه مثل ایکس و ایگرد

davy jones
04-10-2009, 14:16
حتما متغیر ها هستن دیگه مثل ایکس و ایگرد

خب حالا باید چی حل بشه؟

attractive_girl
04-10-2009, 18:59
فکر میکنم یه جورایی از اون معماهای LOVE MATH باشه !

احتمالا I یعنی من و U یعنی تو !
خودمم نمیدونم چجوریه!

k1khansalam
04-10-2009, 21:13
ثابت کنید برای هر عدد طبیعی ایکس که مجذور کامل نباشد رادیکال ایکس گنگ است

Rasam 360
09-10-2009, 21:39
سلام
یه مقدار آماتوریه اما به هر حال : با داشتن اندازه دو ضلع از مثلث و زاویه بین آنها اندازه ضلع سوم چطور محاسبه میشه ؟

singleguy
09-10-2009, 21:51
سلام بر اساتید ریاضی:11:
دو تا سوال کودکانه دارم!:20:
حد چپ و راست تابع زیر در نقطه ی صفر چی میشه؟!

[ برای مشاهده لینک ، لطفا با نام کاربری خود وارد شوید یا ثبت نام کنید ]


و حاصل عبارت زیر:


[ برای مشاهده لینک ، لطفا با نام کاربری خود وارد شوید یا ثبت نام کنید ]

Paradise_human
09-10-2009, 23:39
سلام یه سوال خیلی ساده دارم اگه ممکنه کسی کمکم کنه .
میخوام مشتق (sec(x رو ثابت کنم .
از اونجایی که مثلثاتم خیلی قویه نمیتونم موقع رفع ابهام اونو به به عبارت مثلثاتی دیگه ای تبدیل کنم .
البته میدونم خودش میشه 1 تقسیم بر cosx ولی نمیدنم چطور رفع ابهامش کنم.
اگه ممکنه کمک کنید.
یه سوال دیگه هم باز در رابطه با مشتق دارم .....
یه تابع دو ضابطه ای دارم که میخوام مشتقشو توی نقطه ی صفر بررسی کنم .
تابع این هستش :
یه ضابطه اش (sin (1/x و یه ضابطه اش صفره .
اگه ممکنه توی رفع ابهام اون ضابطه مثلثاتی هم کمک کنید.
ممنون.

amintnt
12-10-2009, 11:01
سلام
یه مقدار آماتوریه اما به هر حال : با داشتن اندازه دو ضلع از مثلث و زاویه بین آنها اندازه ضلع سوم چطور محاسبه میشه ؟
a^2+b^2-2abcosx

سلام بر اساتید ریاضی:11:
دو تا سوال کودکانه دارم!:20:
حد چپ و راست تابع زیر در نقطه ی صفر چی میشه؟!

[ برای مشاهده لینک ، لطفا با نام کاربری خود وارد شوید یا ثبت نام کنید ]



و حاصل عبارت زیر:





[ برای مشاهده لینک ، لطفا با نام کاربری خود وارد شوید یا ثبت نام کنید ]








سلام...

در مورد اولي... خوب در اطراف صفر از ضابطه ي اول بايد استفاده کرد... چون x بين 1- و 1 هست... حد چپ و راست ميشه 1

در مورد سوال دوم، از اون جايي که من هرجا عدد نپر رو ميبينم بر و بر به سوال نگاه ميکنم، جوابي ندارم!

سلام یه سوال خیلی ساده دارم اگه ممکنه کسی کمکم کنه .
میخوام مشتق (sec(x رو ثابت کنم .
از اونجایی که مثلثاتم خیلی قویه نمیتونم موقع رفع ابهام اونو به به عبارت مثلثاتی دیگه ای تبدیل کنم .
البته میدونم خودش میشه 1 تقسیم بر cosx ولی نمیدنم چطور رفع ابهامش کنم.
اگه ممکنه کمک کنید.
یه سوال دیگه هم باز در رابطه با مشتق دارم .....
یه تابع دو ضابطه ای دارم که میخوام مشتقشو توی نقطه ی صفر بررسی کنم .
تابع این هستش :
یه ضابطه اش (sin (1/x و یه ضابطه اش صفره .
اگه ممکنه توی رفع ابهام اون ضابطه مثلثاتی هم کمک کنید.
ممنون.
فرمول مشتق رو که نوشتي، صورت کسر رو در نظر بگير... حالا مخرج مشترک بگير... ميشه Cosa-Cosx که طبق اون چه در حسابان خونديم ميدونيم:
Cosa-Cosx= -1/2Sin[(a+x)/2] *Sin[(a-x)/2]l..... حالا در مورد سينوس دومي از هم ارزي استفاده کن... بعد با مخرج ساده ميشه و جاي x مقدار a رو قرار بده که ميرسي به secx.tanx

در مورد دومي... مشتق وجود نداره... چون از اطراف صفر که بخواي نزديک بشي مقدار سينوس متغير هست و عدد ثابتي نيست....

davy jones
12-10-2009, 13:50
ثابت کنید برای هر عدد طبیعی ایکس که مجذور کامل نباشد رادیکال ایکس گنگ است

اثباتش ساده اس. از برهان خلف استفاده کن و فرض کن که گویا باشه. پس یه کسر گویا مبشه به اون نسبت داد که صورت و مخرج اون اعداد صحیح هستند و عامل مشترکی هم ندارن. اگه عامل مشترک بین صورت و مخرج باشه، کسر رو ساده میکنیم تا دیگه عامل مشترکی نداشته باشه. بعد کسر رو به توان 2 میرسونیم و مطمئنیم که هنوز صورت و مخرج که حالا به توان دو رسیدن، عدد صحیح هستند. بنابراین کسر جدید هم گویاست که این با فرض قضیه که عدد اولیه گویا نیست در تناقضه.

موفق باشین.

davy jones
12-10-2009, 13:58
سلام
یه مقدار آماتوریه اما به هر حال : با داشتن اندازه دو ضلع از مثلث و زاویه بین آنها اندازه ضلع سوم چطور محاسبه میشه ؟

این سوال در حد دوم دبیرستانه!
ولی از اون جایی که شاید شما کوچیکتر از دوم دبیرستان باشی باید به خدمت انور و عالیه شما عارض بشم که:
اگر به این دو ضلع مثلث به چشم دو بردار نگاه کنیم که ابتدای آنها منطبق بر هم و بین آنها زاویه x وجود داره ضلع سوم مثلث در حکم تفاضل برداری این دو برداره. که طبق قانون کسینوسها اندازه بردار سوم به توان 2 برابر است با مجموع مربعات دو ضلع اول منهای دو برابر حاصلضرب اندازه دو ضلع ضربدر cos x است.
ببخشید که حالشو ندارم بطور ریاضی تایپ کنم.

موفق باشین.

a.g g.a
12-10-2009, 20:35
سلام
سوال من نسبت به سوال هاي ديگران خيلي بچگونه به نظرميرسه ولي من تازه کلاس اول دبيرستانم واين سوال خيلي ذهن منو مشغول کرده البته اين يک سوال نيست که قرار باشه حلش کرد شايد هم جاش تو اين تايپيک نباشه
اين چه طور امکان دارد؟
[ برای مشاهده لینک ، لطفا با نام کاربری خود وارد شوید یا ثبت نام کنید ]

zahedy2006
12-10-2009, 20:56
سلام
سوال من نسبت به سوال هاي ديگران خيلي بچگونه به نظرميرسه ولي من تازه کلاس اول دبيرستانم واين سوال خيلي ذهن منو مشغول کرده البته اين يک سوال نيست که قرار باشه حلش کرد شايد هم جاش تو اين تايپيک نباشه
اين چه طور امکان دارد؟


اتفاقا سوال جالبي هم هست

0.3 با دوره گردش را مي توان نوشت
[ برای مشاهده لینک ، لطفا با نام کاربری خود وارد شوید یا ثبت نام کنید ]{3}{10}+\frac{3}{100}+\frac{3}{1000 }+\frac{3}{10000}+...

كه ميشه زيرين

[ برای مشاهده لینک ، لطفا با نام کاربری خود وارد شوید یا ثبت نام کنید ] \frac{3}{10^{n}}

كه حد بي نهايتش همون 3/1 ميشه (البته يه خورده محاسبات داشت كه يادم نيست!)

amintnt
13-10-2009, 12:33
سلام
سوال من نسبت به سوال هاي ديگران خيلي بچگونه به نظرميرسه ولي من تازه کلاس اول دبيرستانم واين سوال خيلي ذهن منو مشغول کرده البته اين يک سوال نيست که قرار باشه حلش کرد شايد هم جاش تو اين تايپيک نباشه
اين چه طور امکان دارد؟
[ برای مشاهده لینک ، لطفا با نام کاربری خود وارد شوید یا ثبت نام کنید ]

در واقع درست هست... شما در تساوي اول که 3/1 رو مساوي 0.3 با دوره ي گردش ميگيرين، در واقع مقدار دقيقي رو بيان نميکنيد... مثلا از شما بپرسن بعد از مميز چندتا 3 هست، آيا ميتونيد تعداد دقيقشو بيان کنيد؟ اگه نخواين که 0.9 با دوره ي گردش رو مساوي 1 در نظر بگيريد، پس در ابتدا هم نبايد 3/1 رو مساوي 0.3 با دوره ي گردش در نظر بگيريد...

اتفاقا سوال جالبي هم هست

0.3 با دوره گردش را مي توان نوشت
[ برای مشاهده لینک ، لطفا با نام کاربری خود وارد شوید یا ثبت نام کنید ]{3}{10}+\frac{3}{100}+\frac{3}{1000 }+\frac{3}{10000}+...

كه ميشه زيرين

[ برای مشاهده لینک ، لطفا با نام کاربری خود وارد شوید یا ثبت نام کنید ] \frac{3}{10^{n}}

كه حد بي نهايتش همون 3/1 ميشه (البته يه خورده محاسبات داشت كه يادم نيست!)
بله درست ميفرماييد... در ادامه ي پست شما يه تصاعد هندسي نزولي ميشه با قدر نسبت 0.1 که با استفاده از فرمولش به همون 3/1 ميرسيم... جمله ي اول به روي يک منهاي قدر نسبت

sherlockholmz
13-10-2009, 16:20
سلام
سوال من نسبت به سوال هاي ديگران خيلي بچگونه به نظرميرسه ولي من تازه کلاس اول دبيرستانم واين سوال خيلي ذهن منو مشغول کرده البته اين يک سوال نيست که قرار باشه حلش کرد شايد هم جاش تو اين تايپيک نباشه
اين چه طور امکان دارد؟
[ برای مشاهده لینک ، لطفا با نام کاربری خود وارد شوید یا ثبت نام کنید ]

باسلام
[ برای مشاهده لینک ، لطفا با نام کاربری خود وارد شوید یا ثبت نام کنید ] 5Cbar%7B3%7D%5CRightarrow%2010x-x=3.%5Cbar%7B3%7D-0.%5Cbar%7B3%7D%5CRightarrow%209x=3%5CRightarrow%2 0x=3/9=1/3%5CRightarrow%200.%5Cbar%7B3%7D=1/3

موفق باشيد

eh_mn
13-10-2009, 19:27
فرض كنيد [ برای مشاهده لینک ، لطفا با نام کاربری خود وارد شوید یا ثبت نام کنید ]{p_1,p_2,\dots\} مجموعه‌ي اعداد اول باشد. در اين صورت در مورد همگرايي و واگرايي سري زير چه مي‌توان گفت؟


[ برای مشاهده لینک ، لطفا با نام کاربری خود وارد شوید یا ثبت نام کنید ]{n=1}^\infty\frac{1}{p_n}

mahsa1469
15-10-2009, 10:41
نشان دهید f(z)=zIm(z)فقط در نقطه ی z=0 مشتق پذیر است!! مشتق را در این نقطه به دست آورید

Alex merccer
16-10-2009, 16:28
سلام به همگی
یه سوال داشتم (در واقع واسه کسی می خوام)
بزرگترین عدد اول که تا حالا کشف شده چیه چند رقم داره؟
یک سوال دیگه هم دارم چرابیشتر اعداد اول خیلی بزرگ که تا حالا کشف شدن جزو اعداد مرسن هستند و اعداد اولی که جزو اعداد مرسن نباشن بینشون دیده نمی شه؟
ممنون

davy jones
17-10-2009, 11:16
سلام.
این سوال رو تو المپیاد دانش آموزی ریاضی دیدم. سوال اینه:
فرض کنید قورباغه ای روی نقطه (0،1) در محور مختصات ( یعنی روی محور y ها) ایستاده باشد. این قورباغه در هر بار پرش خود ، در راستای عمود بر خطی که مکان فعلی وی را به مبدا وصل میکند ، به اندازه ی طول همان خط واصل پرش میکند.( یعنی در پرش اول به اندازه 1 واحد به سمت راست جهش کرده و به نقطه ی (1،1) میرسد). بعد از 15 بار پرش این قور باغه ، عرض جغرافیایی مختصات آن چقدر است؟

موفق باشین.
88/7/24

Paradise_human
17-10-2009, 12:07
فرمول مشتق رو که نوشتي، صورت کسر رو در نظر بگير... حالا مخرج مشترک بگير... ميشه Cosa-Cosx که طبق اون چه در حسابان خونديم ميدونيم:
Cosa-Cosx= -1/2Sin[(a+x)/2] *Sin[(a-x)/2]l..... حالا در مورد سينوس دومي از هم ارزي استفاده کن... بعد با مخرج ساده ميشه و جاي x مقدار a رو قرار بده که ميرسي به secx.tanx

آقا این هم ارزی یه خورده بیشتر توضیخ میدی ؟
چطور باید از هم ارزی استفاده کنم ؟

amintnt
17-10-2009, 12:21
آقا این هم ارزی یه خورده بیشتر توضیخ میدی ؟
چطور باید از هم ارزی استفاده کنم ؟
از اين هم ارزي:
Sinu وقتي u به سمت 0 ميل ميکنه برابر ميشه با u...

اينجا هم Sin[(a-x)/2]l چون x به a ميل ميکنه، مقدار کمان به صفر ميل ميکنه، بنابر اين برابر ميشه با کمان، يعني a-x به روي 2...

Desmond
18-10-2009, 18:00
ثابت کنید دنباله {[ برای مشاهده لینک ، لطفا با نام کاربری خود وارد شوید یا ثبت نام کنید ]} با جمله عمومی
[ برای مشاهده لینک ، لطفا با نام کاربری خود وارد شوید یا ثبت نام کنید ] 1%7D
که در آن [ برای مشاهده لینک ، لطفا با نام کاربری خود وارد شوید یا ثبت نام کنید ]،همگرا به صفر است.

Desmond
18-10-2009, 18:08
ثابت کنید که دنباله [ برای مشاهده لینک ، لطفا با نام کاربری خود وارد شوید یا ثبت نام کنید ] 20%5C%7D با جمله عمومی

[ برای مشاهده لینک ، لطفا با نام کاربری خود وارد شوید یا ثبت نام کنید ] %7D%201/n%20&,n=2k-1%20%5C%5Cn/%28n+2%29%20&,n=2k%20%5Cend%7Bmatrix%7D%5Cright.

دارای حد نمی باشد.

Desmond
18-10-2009, 18:11
آیا دنباله [ برای مشاهده لینک ، لطفا با نام کاربری خود وارد شوید یا ثبت نام کنید ] ight%20%5C%7D همگراست؟چرا؟

Desmond
18-10-2009, 18:15
ثابت کنید دنباله [ برای مشاهده لینک ، لطفا با نام کاربری خود وارد شوید یا ثبت نام کنید ] 20%5C%7D که در آن [ برای مشاهده لینک ، لطفا با نام کاربری خود وارد شوید یا ثبت نام کنید ] و [ برای مشاهده لینک ، لطفا با نام کاربری خود وارد شوید یا ثبت نام کنید ] کراندار و صعودی است و سپس حد آن را به دست آورید.

singleguy
20-10-2009, 18:48
سلام
یه سوال از درس ریاضی مهندسی_ معادله موج دارم
من دنبال u(x,t) l میگردم. رفتم جولو تا در انتخاب C1 , C2 گیر کردم! اگه میشه کمکم کنید!
این هم اون قسمتی که خودم رفتم: (ببخشید اگه خطم بده)

برای مشاهده محتوا ، لطفا وارد شوید یا ثبت نام کنید

پیشاپیش ممنونم [ برای مشاهده لینک ، لطفا با نام کاربری خود وارد شوید یا ثبت نام کنید ]

kwpa
24-10-2009, 10:25
سلام به دوستان

دیروز یه بچه راهنمایی سوالی رو مطرح کرد که من نتونستم جواب بدم. ممکنه راهنماییم کنین:

اگه 3 تا کارگر A، B و C داشته باشیم به ترتیبی که دوبه دو با هم کار کنن:
AوB با هم X روز، Bو C با هم Y روز، Aو C با هم Z روز
و داشته باشیم که کارگر A ضریبی از X رو کار میکنه ( mX روز ) و به همین ترتیب: کارگر B ضریبی از Y رو ( nY روز ) و همچنین کارگر C ضریبی از Z رو (pY روز )

آنگاه چه رابطه مستقلی از X، Y،Z بین m،n و P ,وجود داره؟

ممنونم

bn_ha
24-10-2009, 18:54
ببخشید دوستان من سوالی دارم.:
درصورتی که تابع F پیوسته روی بازه ی[a,b] باشد و روی بازه ی (a,b) مشتق پذیر باشد.همچنین F (a = F(b آنگاه ثابت کنید عددی مانند c وجود دارد که مشتق تابع در c مساوی F(c می شود.

bn_ha
24-10-2009, 18:56
ثابت کنید دنباله {[ برای مشاهده لینک ، لطفا با نام کاربری خود وارد شوید یا ثبت نام کنید ]} با جمله عمومی
[ برای مشاهده لینک ، لطفا با نام کاربری خود وارد شوید یا ثبت نام کنید ] 1%7D
که در آن [ برای مشاهده لینک ، لطفا با نام کاربری خود وارد شوید یا ثبت نام کنید ]،همگرا به صفر است.

سلام.
با توجه به این که سرعت رشد مخرج از صورت بیشتره پس حد آن به سمت بی نهایت می شه صفر

bn_ha
24-10-2009, 18:57
آیا دنباله [ برای مشاهده لینک ، لطفا با نام کاربری خود وارد شوید یا ثبت نام کنید ] ight%20%5C%7D همگراست؟چرا؟
خیر چون حد آن را که به سمت بی نهایت می گیریم می شه بی نهایت پس بی کران است در نتیجه واگراست

MCElectronics
24-10-2009, 21:23
سلام
1 از دوستانم از من خواست تا 1 سوال ریاضی رو براش حل کنم
میشه کمک کنید

سوال اینه:
[ برای مشاهده لینک ، لطفا با نام کاربری خود وارد شوید یا ثبت نام کنید ]


معادله نزدیکترین خط راستی را که از این نقاط میگذرد؟

mehdi_7070
24-10-2009, 21:35
سلام

می‌خواهیم که بسط مک لورن تابع [ برای مشاهده لینک ، لطفا با نام کاربری خود وارد شوید یا ثبت نام کنید ] را بنویسیم. محاسبات را انجام میدیم تا برسیم به [ برای مشاهده لینک ، لطفا با نام کاربری خود وارد شوید یا ثبت نام کنید ] D

حالا برای محاسبه مقدار [ برای مشاهده لینک ، لطفا با نام کاربری خود وارد شوید یا ثبت نام کنید ] و سایر مشتقات می‌تونیم از بسط مک لورن [ برای مشاهده لینک ، لطفا با نام کاربری خود وارد شوید یا ثبت نام کنید ] که می‌شه [ برای مشاهده لینک ، لطفا با نام کاربری خود وارد شوید یا ثبت نام کنید ] استفاده کنیم؟ مثلا:

[ برای مشاهده لینک ، لطفا با نام کاربری خود وارد شوید یا ثبت نام کنید ]


ممنون:11:

ali_hp
24-10-2009, 22:02
ببخشید دوستان من سوالی دارم.:
درصورتی که تابع F پیوسته روی بازه ی[a,b] باشد و روی بازه ی (a,b) مشتق پذیر باشد.همچنین F (a = F(b آنگاه ثابت کنید عددی مانند c وجود دارد که مشتق تابع در c مساوی F(c می شود.
سلام
سوال درست نیست،مثلا تابع ثابت F(x)=1 را در بازه [3,4] در نظر بگیرید.

roham_06
25-10-2009, 09:50
سلام....آقا ما توي اين اتگرال گير كرديم!!!
نميدونم چطوري اينو نتيجه گرفته....
اينم عكسش::::
[ برای مشاهده لینک ، لطفا با نام کاربری خود وارد شوید یا ثبت نام کنید ]

bn_ha
25-10-2009, 11:20
سلام
سوال درست نیست،مثلا تابع ثابت F(x)=1 را در بازه [3,4] در نظر بگیرید.
ببخشید 1 شرط رو یادم رفته بود بگذارم
F(a = F(b = 0

davy jones
25-10-2009, 16:08
ببخشید 1 شرط رو یادم رفته بود بگذارم
F(a = F(b = 0

بازم سوال درست نیست. برای اینکه شیر فهم بشی این نکته رو همیشه بیاد داشته باش که تنها تابع غیر صفری که مشتقش با خودش برابره تابع نماییه:
y=e^x
حالا اگه تابع f با همه ی اون مشخصاتی که گفتی با این تابع حداقل یه بار برخورد کنه مساله جواب داره ولی خیلی واضحه که میشه یه تابع f مثال زد که در یک بازه (a,b) که a,b>0 هستند پیوسته و مشتقپذیر باشه و f(a)=f(b)=0 هم صادق باشه ولی با نمودار تابع نمایی تداخل نداشته باشه.
احتمالا سوال یه شرط دیگه هم داره مثلا اینکه: a<0
امیدوارم تونسته باشم منظورمو درست بگم.

موفق باشین.
88/8/3

davy jones
25-10-2009, 16:11
آیا دنباله [ برای مشاهده لینک ، لطفا با نام کاربری خود وارد شوید یا ثبت نام کنید ] ight%20%5C%7D همگراست؟چرا؟

معلومه که نیست. چون رادیکال n یه تابع اکیدا صعودیه.

موفق باشین.
88/8/3

davy jones
25-10-2009, 16:23
سلام
1 از دوستانم از من خواست تا 1 سوال ریاضی رو براش حل کنم
میشه کمک کنید

سوال اینه:
[ برای مشاهده لینک ، لطفا با نام کاربری خود وارد شوید یا ثبت نام کنید ]


معادله نزدیکترین خط راستی را که از این نقاط میگذرد؟


من حوصله ی حساب کردنشو ندارم. ولی راهنمایی میکنم که یه معادله ی کلی مثل:
y=ax+b
در نظر بگیر و مجموع مربعات طول پاره خطهای عمود بر این تابع رو برای تک تک داده ها بطور پارامتری حساب میکنی. حالا این باید مینیمم بشه. یه بار نسبت به a و یه بار نسبت به b مشتق میگیری و اونا رو مساوی با صفر میذاری تا a و b مشخص بشن.

البته یه راه بسیار ساده تر هم داره اونم اینکه بری توی محیط excel و این داده ها رو وارد میکنی و میری تو قسمت نمودار کشیدن و توی اختیارات اضافه ی نمودار میتونی تعیین کنی که خود excel این کار رو بکنه. معادله خط و واریانس رو هم خودش حساب میکنه و بهت میگه.

موفق باشین.
88/8/3

bn_ha
25-10-2009, 18:39
بازم سوال درست نیست. برای اینکه شیر فهم بشی این نکته رو همیشه بیاد داشته باش که تنها تابع غیر صفری که مشتقش با خودش برابره تابع نماییه:
y=e^x
حالا اگه تابع f با همه ی اون مشخصاتی که گفتی با این تابع حداقل یه بار برخورد کنه مساله جواب داره ولی خیلی واضحه که میشه یه تابع f مثال زد که در یک بازه (a,b) که a,b>0 هستند پیوسته و مشتقپذیر باشه و f(a)=f(b)=0 هم صادق باشه ولی با نمودار تابع نمایی تداخل نداشته باشه.
احتمالا سوال یه شرط دیگه هم داره مثلا اینکه: a<0
امیدوارم تونسته باشم منظورمو درست بگم.

موفق باشین.
88/8/3
شما 1 مثال نقض برای من بیار.منظور من از سوال این هست که حد اقل 1 سی وجود دارد که در شرایط مسئله صدق می کند.

ali_hp
25-10-2009, 22:11
ببخشید 1 شرط رو یادم رفته بود بگذارم
F(a = F(b = 0
با این شرط حکم برقراره...
اثبات:
طبق قضیه رل نقطه ای مثل w وجود دارد که مشتق f در w صفر شود و a<w<b .اگر f(w)=0 که w همان نقطه مورد نظر است.پس بدون کم شدن از کلیت مساله فرض کنید f(w)>0 .قرار دهید :

[ برای مشاهده لینک ، لطفا با نام کاربری خود وارد شوید یا ثبت نام کنید ]{&space;t&space;|&space;a\le{t}<w&space;,&space;f(t)=0\}

با توجه به پیوستگی f داریم f(S)=0.اگر مشتق f در S صفر باشد که S همان نقطه مورد نظر است. همچنین اگر مشتق f در S منفی باشد،با sup بودن S متناقض است.پس مشتق f در S مثبت است،حال تابع

[ برای مشاهده لینک ، لطفا با نام کاربری خود وارد شوید یا ثبت نام کنید ](x)-f^{\prime}(x) را در بازه [S,w] در نظر بگیرید،در S منفی است و در w مثبت.پس با توجه به قضیه مقدار میانی در نقطه ای مثل c صفر می شود.

ali_hp
25-10-2009, 22:20
معلومه که نیست. چون رادیکال n یه تابع اکیدا صعودیه.

موفق باشین.
88/8/3
سلام،این دنباله همگرا نیست چون بیکرانه...
اگر یک دنباله اکیدا صعودی باشه نمیشه نتیجه گرفت که همگرا نیست.مثلا دنباله

[ برای مشاهده لینک ، لطفا با نام کاربری خود وارد شوید یا ثبت نام کنید ]{1}{n}

اکیدا صعودیه ولی همگرا است.

davy jones
26-10-2009, 08:37
شما 1 مثال نقض برای من بیار.منظور من از سوال این هست که حد اقل 1 سی وجود دارد که در شرایط مسئله صدق می کند.

درسته! اشتباه از من بود. ممنون.

davy jones
26-10-2009, 08:39
سلام،این دنباله همگرا نیست چون بیکرانه...
اگر یک دنباله اکیدا صعودی باشه نمیشه نتیجه گرفت که همگراست.مثلا دنباله

[ برای مشاهده لینک ، لطفا با نام کاربری خود وارد شوید یا ثبت نام کنید ]

اکیدا صعودیه ولی همگرا نیست.

من هم نتیجه نگرفتم که همگراست. من گفتم چون اکیدا صعودیه پس واگراست.

amintnt
26-10-2009, 10:21
من هم نتیجه نگرفتم که همگراست. من گفتم چون اکیدا صعودیه پس واگراست.

اگر یک دنباله اکیدا صعودی باشه نمیشه نتیجه گرفت که همگراست.مثلا دنباله

[ برای مشاهده لینک ، لطفا با نام کاربری خود وارد شوید یا ثبت نام کنید ]{1}{n}

اکیدا صعودیه ولی همگرا نیست.
فکر ميکنم جاي "نيست" و "است" بايد عوض بشه...



من گفتم چون اکیدا صعودیه پس واگراست

البته خودتون بهتر ميدونيد... اما بايد شرط کراندار بودن هم برقرار باشه....



سلام....آقا ما توي اين اتگرال گير كرديم!!!
نميدونم چطوري اينو نتيجه گرفته....
اينم عكسش::::
[ برای مشاهده لینک ، لطفا با نام کاربری خود وارد شوید یا ثبت نام کنید ]


انتگرالي که گرفته صحيح هست ديگه... اون t هم که از انتگرال طرف دوم محاسبه شده... به کجاش مشکوکي شما؟

ali_hp
26-10-2009, 10:21
من هم نتیجه نگرفتم که همگراست. من گفتم چون اکیدا صعودیه پس واگراست.
سلام
اشتباه نوشتم!ممنونم از تذکرتون.
منم منظورم همین بود که نمیشه نتیجه گرفت واگراست!دنباله ای که مثال زدم اکیدا صعودی ولی واگرا نیست!پستمو ویرایش کردم!

4346304]فکر ميکنم جاي "نيست" و "است" بايد عوض بشه...[/u]

بله،باید عوض بشه،متشکرم

ali_hp
26-10-2009, 10:55
سلام

می‌خواهیم که بسط مک لورن تابع [ برای مشاهده لینک ، لطفا با نام کاربری خود وارد شوید یا ثبت نام کنید ] را بنویسیم. محاسبات را انجام میدیم تا برسیم به [ برای مشاهده لینک ، لطفا با نام کاربری خود وارد شوید یا ثبت نام کنید ] D

حالا برای محاسبه مقدار [ برای مشاهده لینک ، لطفا با نام کاربری خود وارد شوید یا ثبت نام کنید ] و سایر مشتقات می‌تونیم از بسط مک لورن [ برای مشاهده لینک ، لطفا با نام کاربری خود وارد شوید یا ثبت نام کنید ] که می‌شه [ برای مشاهده لینک ، لطفا با نام کاربری خود وارد شوید یا ثبت نام کنید ] استفاده کنیم؟ مثلا:

[ برای مشاهده لینک ، لطفا با نام کاربری خود وارد شوید یا ثبت نام کنید ]


ممنون:11:
بله،می تونید استفاده کنید.در حقیقت مشتق یک تابع در بازه ای که بسط مک لورن آن تابع،با آن برابر می شود،برابر سری می شود که از مشتق گیری جمله به جمله بسط مک لورن تابع بدست می آید.

ali_hp
26-10-2009, 11:17
ثابت کنید که دنباله [ برای مشاهده لینک ، لطفا با نام کاربری خود وارد شوید یا ثبت نام کنید ] 20%5C%7D با جمله عمومی

[ برای مشاهده لینک ، لطفا با نام کاربری خود وارد شوید یا ثبت نام کنید ] %7D%201/n%20&,n=2k-1%20%5C%5Cn/%28n+2%29%20&,n=2k%20%5Cend%7Bmatrix%7D%5Cright.

دارای حد نمی باشد.
این دنباله دارای دو زیر دنباله است که یکی به یک میل می کند،و دیگری به صفر پس با توجه به لم زیر نمی تواند همگرا باشد.
لم:شرط لازم برای همگرایی دنباله a_n این است که:

[ برای مشاهده لینک ، لطفا با نام کاربری خود وارد شوید یا ثبت نام کنید ]{n\rightarrow\infty}{|a_n-a_{n-1}|}=0
یعنی تفاضل جملات متوالی به صفر میل کند.

ali_hp
26-10-2009, 11:29
ثابت کنید دنباله [ برای مشاهده لینک ، لطفا با نام کاربری خود وارد شوید یا ثبت نام کنید ] 20%5C%7D که در آن [ برای مشاهده لینک ، لطفا با نام کاربری خود وارد شوید یا ثبت نام کنید ] و [ برای مشاهده لینک ، لطفا با نام کاربری خود وارد شوید یا ثبت نام کنید ] کراندار و صعودی است و سپس حد آن را به دست آورید.
با استفاده از استقرا ثابت کنید که a_n<4 و صعودی بودن رانیز با استفاده از استقرا ثابت کنید.

و دقت کنید که حد دنباله باید در معادله [ برای مشاهده لینک ، لطفا با نام کاربری خود وارد شوید یا ثبت نام کنید ]{x&plus;12} صدق کند.

singleguy
27-10-2009, 22:02
سلام عزیزان!
این دوتا انتگ را برام اگه میشه زحمتشو بکشید!:

[ برای مشاهده لینک ، لطفا با نام کاربری خود وارد شوید یا ثبت نام کنید ]

afshin b
27-10-2009, 23:04
سلام عزیزان!
این دوتا انتگ را برام اگه میشه زحمتشو بکشید!:

[ برای مشاهده لینک ، لطفا با نام کاربری خود وارد شوید یا ثبت نام کنید ]

من اولين بارمه تو اين قسمت ميام بلد نيستم بصورت رياضي بنويسم(اگه ميشه ياد بدين):41:
با اجازه اساتيد
انتگرال اول با دوبار جز به جز حل ميشه. جواب:
2/(e^x (sinx-cosx
دومي هم صورت و مخرجو در e^-x ضرب ميكنيم اگه مخرجو u فرض كنيم du رو راحت ميشه تو صورت ايجاد كرد و جواب ميشه:
(Ln(e^x /1+e^x

mehdi_7070
28-10-2009, 09:46
من اولين بارمه تو اين قسمت ميام بلد نيستم بصورت رياضي بنويسم(اگه ميشه ياد بدين):41:


انتشار درست عبارات و فرمول های ریاضی در سایت ([ برای مشاهده لینک ، لطفا با نام کاربری خود وارد شوید یا ثبت نام کنید ])

Paradise_human
30-10-2009, 20:54
سلام.
دو تا سوال خیلی ساده دارم.
معادله ی خط مماس بر منحنی به معادله ی :
4x^2--y^2+16x+2y+11=0
را بنویسید که موازی با محور عرض ها باشد.
منظور از اینکه موازی با مخور عرض ها باشد یعنی چی ؟
و سوال دوم :
از نقطه ی A به مختصات (P/2 ,0) منظورم از P عدد پی هستش خطی بر معادله ی :
(y^3 =sin(x-y
رسم شده است.مطلوب است معادله ی خط مماس.

bn_ha
31-10-2009, 00:28
سلام.
دو تا سوال خیلی ساده دارم.
معادله ی خط مماس بر منحنی به معادله ی :
4x^2--y^2+16x+2y+11=0
را بنویسید که موازی با محور عرض ها باشد.
منظور از اینکه موازی با مخور عرض ها باشد یعنی چی ؟
و سوال دوم :
از نقطه ی A به مختصات (P/2 ,0) منظورم از P عدد پی هستش خطی بر معادله ی :
(y^3 =sin(x-y
رسم شده است.مطلوب است معادله ی خط مماس.
سلام
در مورد سوال اول:منظورش این هست که با محور ایکس ها موازی باشه .محور ایکس ها هم که افقی هست و شیبش صفره پس شما باید مشتق ضمنی گرفته و اون رو مساوی صفر قرار داده .

davy jones
31-10-2009, 10:17
معادله ی خط مماس بر منحنی به معادله ی :
4x^2--y^2+16x+2y+11=0
را بنویسید که موازی با محور عرض ها باشد.
منظور از اینکه موازی با مخور عرض ها باشد یعنی چی ؟
منظورش این هست که با محور ایکس ها موازی باشه .سلام به شما دوستان. منظور از محور عرض ها، محور y هاست و به محور x ها، محور طول میگن. شما باید مشتق رو در جایی که به سمت بینهایت میل میکنه محاسبه کنی. در حقیقت این معادله بسط داده شده ی یک هذلولی دوران یافته در صفحه xy است.
4x^2-y^2+16x+2y+11
=4(x^2+4x+4-4)-(y^2-2y+1-1)+11
=4(x+2)^2-16-(y-1)^2+1+11=0
(x+2)^2-(y-1)^2/4=1

پس مرکز هذلولی نقطه x=-2 و y=1 است.

موفق باشین.
88/8/9

CATALONIA
31-10-2009, 18:29
اگه میشه این سوالات را با تمام جوابشو بنویسید 
ممنون
چند عدد سه رقمی بدون تکرار رقم با ارقام 5 و 4 و 3 و 2 و 0 میتوان نوشت که زوج باشد ؟
باقیمانده ی تقسیم عدد m= ( سه به توان ده + سه به توان دوازده + ...+ سه به توان پنجاه ) بر 899 را به دست آورید ؟

H.R@Wolf
01-11-2009, 15:35
اگه میشه این سوالات را با تمام جوابشو بنویسید
ممنون
چند عدد سه رقمی بدون تکرار رقم با ارقام 5 و 4 و 3 و 2 و 0 میتوان نوشت که زوج باشد ؟
باقیمانده ی تقسیم عدد m= ( سه به توان ده + سه به توان دوازده + ...+ سه به توان پنجاه ) بر 899 را به دست آورید ؟

سلام به دوستان
در مورد سوال اول شما بايد از اصل ضرب و اصل جمع استفاده كني.شما بايد 2 حالت در نظر بگيري يعني فرض كني كه يكان عدد 0 باشد يا يكان عدد 2و4 باشد .بعد تعداد حالات رو با هم جمع ميزني جواب به دست مياد
كه اگه اين كارو انجام بدي، جواب آخر 21 ميشه .:10::20::31:
سوال دوم رو نميدونم بايد فكر كنم .

davy jones
01-11-2009, 15:49
اگه میشه این سوالات را با تمام جوابشو بنویسید
ممنون
چند عدد سه رقمی بدون تکرار رقم با ارقام 5 و 4 و 3 و 2 و 0 میتوان نوشت که زوج باشد ؟
باقیمانده ی تقسیم عدد m= ( سه به توان ده + سه به توان دوازده + ...+ سه به توان پنجاه ) بر 899 را به دست آورید ؟

در مورد سوال اول، سوال رو به سه حالت تقسيم ميكنيم:
1- رقم يكان صفر باشه.............2- رقم يكان 2 باشه.................3- رقم يكان 4 باشه

1 : براي رقم دهگان 4 انتخاب و براي صدگان 3 انتخاب داريم. پس در كل ميشه 12 حالت.

2 : براي صدگان 3 انتخاب و براي دهگان هم 3 انتخاب داريم.(صفر نميتونه در رقم صدگان باشه چون ديگه عدد ما سه رقمي نميشه) پس ميشه در كل 9 حالت.

3 : باز هم مثل حالت 2 در كل 9 انتخاب داريم.

با توجه به اين كه اين حالات از هم مستقل نيستن، پس بايد با هم جمع بشن. پس جواب سوال اول شما برابر با 9+9+12=30 حالت است.

در مورد سوال دوم هم فكر ميكنم كه سوال سر كاريه!

موفق باشين.
88/8/10

CATALONIA
01-11-2009, 16:35
در مورد سوال اول، سوال رو به سه حالت تقسيم ميكنيم:
1- رقم يكان صفر باشه.............2- رقم يكان 2 باشه.................3- رقم يكان 4 باشه

1 : براي رقم دهگان 4 انتخاب و براي صدگان 3 انتخاب داريم. پس در كل ميشه 12 حالت.

2 : براي صدگان 3 انتخاب و براي دهگان هم 3 انتخاب داريم.(صفر نميتونه در رقم صدگان باشه چون ديگه عدد ما سه رقمي نميشه) پس ميشه در كل 9 حالت.

3 : باز هم مثل حالت 2 در كل 9 انتخاب داريم.

با توجه به اين كه اين حالات از هم مستقل نيستن، پس بايد با هم جمع بشن. پس جواب سوال اول شما برابر با 9+9+12=30 حالت است.

  در مورد سوال دوم هم فكر ميكنم كه سوال سر كاريه!


موفق باشين.
88/8/10


ممنون 

ولی فکر نکنم سر کاری باشه 
چون دبیر ریاضیمون داده بهمون 

ali_hp
01-11-2009, 18:21
باقیمانده ی تقسیم عدد m= ( سه به توان ده + سه به توان دوازده + ...+ سه به توان پنجاه ) بر 899 را به دست آورید ؟
سلام
راه حل طولانی و خسته کننده ای داره این مساله و نیازمند اینه که با قضیه کوچک فرما و خواص همنهشتیها تسلط کامل داشته باشید ،من فکر می کنم سوال دقیقا این نباشه،به نظر من تو صورت سوال به جای سه باید عدد سی باشه(یعنی توانهای سی با هم جمع بشن)شما دوباره سوالو با منبعش چک کن ببین دقیقا همینه؟

CATALONIA
03-11-2009, 13:38
سلام
راه حل طولانی و خسته کننده ای داره این مساله و نیازمند اینه که با قضیه کوچک فرما و خواص همنهشتیها تسلط کامل داشته باشید ،من فکر می کنم سوال دقیقا این نباشه،به نظر من تو صورت سوال به جای سه باید عدد سی باشه(یعنی توانهای سی با هم جمع بشن)شما دوباره سوالو با منبعش چک کن ببین دقیقا همینه؟


اگه سی باشه 

راه حلش چطوریه و جوابش چی میشه 

H.R@Wolf
03-11-2009, 14:46
سلام به دوستان
در مورد سوال اول شما بايد از اصل ضرب و اصل جمع استفاده كني.شما بايد 2 حالت در نظر بگيري يعني فرض كني كه يكان عدد 0 باشد يا يكان عدد 2و4 باشد .بعد تعداد حالات رو با هم جمع ميزني جواب به دست مياد
كه اگه اين كارو انجام بدي، جواب آخر 21 ميشه .:10::20::31:
سوال دوم رو نميدونم بايد فكر كنم .
ببخشيد اشتباه كردم جواب 30ميشه .:19::19::2:

H.R@Wolf
03-11-2009, 14:47
سوال:
باقيمانده ي عدد 18 فاكتوريل ضربدر 5 بر عدد 19 چند است؟

mehdi_7070
03-11-2009, 16:16
سوال:
باقيمانده ي عدد 18 فاكتوريل ضربدر 5 بر عدد 19 چند است؟

فکر کنم یه قضیه‌ای هست به نام قضیه ویلسون که اگه n عدد اول باشه، !(n-1) به پیمانه‌ی n همنهشت 1- هست.
که با این حساب جواب 14 می‌شه.

ali_hp
03-11-2009, 17:45
اگه سی باشه

راه حلش چطوریه و جوابش چی میشه
سلام
دقت کنید که 30 به توان دو برابر 900 است،پس باقیمانده 30 به توان دو بر 899 میشود یک!
بنابراین باقیمانده توانهای زوج 30 نیز بر 899 میشود یک!
و در اینجا 21 تا از توانهای زوج سی باهم جمع می شوند،پس جواب نهایی میشه 21!

Mohammad Hosseyn
04-11-2009, 08:57
در مورد سری زیر :

[ برای مشاهده لینک ، لطفا با نام کاربری خود وارد شوید یا ثبت نام کنید ]

اولا جمله ی عمومی اون گویاست یا اسم
دومامجموع سری بالا مقدارش گویا هست یا اصم .

bn_ha
04-11-2009, 15:45
هواپیمایی با تندی ثابت 300 کیلومتر بر ساعت بر فراز یک ایستگاه رادار زمینی درارتفاع 1کیلومتری پرواز کرده و با زاویه ی 30 درجه بالا می رود.1 دقیقه بعد فاصله ی هواپیما تا ایستگاه رادار با چه نرخی در حال افزایش است؟

با عرض پوزش اگه می شه تا امشب جواب بدید چون فردا امتحان دارم.
ممنون

Bibily
04-11-2009, 19:51
سلام کسی می تونه این سوالو حل کنه؟
4تا طناب داریم.میتونیم دوسر هر طنابو به هم بچسبونیم و یه حلقه درست کنیم.میخوایم با این طناب ها 4 حلقه در هم درست کنیم طوری که بدون بریدن این حلقه ها نتوان آن ها را از هم جدا کرد و با بریدن هر کدام بتوان 3حلقه ی دیگر را بدون بریدن از هم جدا کرد

H.R@Wolf
04-11-2009, 22:42
K مهره روي يك ميز در يك دسته قرار دارند . قانون بازي اينگونه است كه در هر حركت، بايد از يك دسته كه بيش از يك مهره دارد،يك مهره دور انداخته شود و سپس يكي از دسته ها كه بيش از يك مهره دارد را به دو بخش ناتهي تقسيم كرد . كوچكترين Kبزرگتر از ۱۳۸۸ را به نحوي بيابيد كه پس از شروع با K مهره و بعد از چند حركت، بتوان به وضعيتي رسيد كه روي ميز تنها دسته هايي با ۳ مهره وجود داشته باشد؟

Paradise_human
06-11-2009, 14:04
سلام.
این معادله ی ساده رو برام حل کنید :
tanA-A/3=-P/6 (منظور از P عدد پی هستش )
ویه سوال دیگه :
با استفاده از قضیه ی مقدار میانگین نشان دهید که اگر 0<b>a آنگاه b-a/a>Ln b/a>b-a/b
ممنون.

Hasan.M
06-11-2009, 17:27
سلام یه سوالAوBروپیدا کنید
2a-b=5
باتشکر arzeva

mahsa1469
06-11-2009, 18:17
خوب a=3و b=1

Iron
08-11-2009, 08:22
سلام کسی می تونه این سوالو حل کنه؟
4تا طناب داریم.میتونیم دوسر هر طنابو به هم بچسبونیم و یه حلقه درست کنیم.میخوایم با این طناب ها 4 حلقه در هم درست کنیم طوری که بدون بریدن این حلقه ها نتوان آن ها را از هم جدا کرد و با بریدن هر کدام بتوان 3حلقه ی دیگر را بدون بریدن از هم جدا کرد


سلام

[ برای مشاهده لینک ، لطفا با نام کاربری خود وارد شوید یا ثبت نام کنید ] ([ برای مشاهده لینک ، لطفا با نام کاربری خود وارد شوید یا ثبت نام کنید ])

Paradise_human
08-11-2009, 17:25
سلام.
این معادله ی ساده رو برام حل کنید :
tanA-A/3=-P/6 (منظور از P عدد پی هستش )
ویه سوال دیگه :
با استفاده از قضیه ی مقدار میانگین نشان دهید که اگر 0<b>a آنگاه b-a/a>Ln b/a>b-a/b
ممنون.
کسی نیست به ما کمک کنه ؟؟
من این جوابارو تا آخر شب میخوام.
اگه امکان داره کمک کنید.
واجبه.
ممنون.

ali_hp
08-11-2009, 18:07
کسی نیست به ما کمک کنه ؟؟
من این جوابارو تا آخر شب میخوام.
اگه امکان داره کمک کنید.
واجبه.
ممنون.
سلام
سوال اولتون واضح نیست.
حل سوال دوم:
از قضیه مقدار میانگین برای تابع ln x در بازه (a,b) استفاده کنید،بنابر این a<c<b وجود دارد که:

[ برای مشاهده لینک ، لطفا با نام کاربری خود وارد شوید یا ثبت نام کنید ]^{\prime}{c}=\frac{1}{c}=\frac{\ln{b}-\ln{a}}{b-a}\\&space;\\&space;\\&space;\Rightarrow\frac{1}{b}<\frac{1}{c}=\frac{\ln{b}-\ln{a}}{b-a}<\frac{1}{a}

که معادل حکم مساله است.

Paradise_human
09-11-2009, 00:26
سلام
سوال اولتون واضح نیست.
حل سوال دوم:
از قضیه مقدار میانگین برای تابع ln x در بازه (a,b) استفاده کنید،بنابر این a<c<b وجود دارد که:

[ برای مشاهده لینک ، لطفا با نام کاربری خود وارد شوید یا ثبت نام کنید ]^{\prime}{c}=\frac{1}{c}=\frac{\ln{b}-\ln{a}}{b-a}\\&space;\\&space;\\&space;\Rightarrow\frac{1}{b}<\frac{1}{c}=\frac{\ln{b}-\ln{a}}{b-a}<\frac{1}{a}

که معادل حکم مساله است.
سئال اولم منظورم اون A معادله رو بدست بیارید.

sh.aut
10-11-2009, 04:30
سلام و خسته نباشین
من یه سولا ریاضی داشتم که اگه جواب بدین کمک خیلی خیلی بزرگی بهم کردین !!!سوال:تابعی مثال بزنید که در تمام نقاط پیوسته باشد ولی در هیچ نقطه ای مشتق پذیر نباشد
اگه ضابطه چنین تابعی رو بگین خیلی ممنون میشم
شیما زینالی
sh_aut.8870@yahoo.com
ممنون

! Time
10-11-2009, 14:36
سلام،
می خواستم بدانم که آیا sin و tan برداری هستند یا نه؟ همراه با دلیل آن؟
متشکرم.

eh_mn
10-11-2009, 21:35
سلام و خسته نباشین
من یه سولا ریاضی داشتم که اگه جواب بدین کمک خیلی خیلی بزرگی بهم کردین !!!سوال:تابعی مثال بزنید که در تمام نقاط پیوسته باشد ولی در هیچ نقطه ای مشتق پذیر نباشد
اگه ضابطه چنین تابعی رو بگین خیلی ممنون میشم
شیما زینالی
sh_aut.8870@yahoo.com
ممنون

با سلام.

تابع معروف به تابع وايرشتراس اين خاصيت رو داره. تعريف مختصري از اين تابع را در

برای مشاهده محتوا ، لطفا وارد شوید یا ثبت نام کنید
يا

برای مشاهده محتوا ، لطفا وارد شوید یا ثبت نام کنید
ملاحظه كنين.

يك انيميشن هم در مورد نمودار اين تابع در

برای مشاهده محتوا ، لطفا وارد شوید یا ثبت نام کنید
وجود داره.

توضيحات كمي فني‌تر رو اينجا ببينيد

برای مشاهده محتوا ، لطفا وارد شوید یا ثبت نام کنید

از سايت‌هاي نارنجي ميشه به عنوان مرجع براي اصطلاحات رياضي استفاده كرد

موفق باشيد.

davy jones
11-11-2009, 13:21
سلام،
می خواستم بدانم که آیا sin و tan برداری هستند یا نه؟ همراه با دلیل آن؟
متشکرم.

فكر نكنم قابل تعريف باشه. چون ديمانسيون آرگومان سينوس و تانژانت، طبق تعريف اونها لزوما بايد راديان باشه و همون طور كه ميدونيد راديان در حقيقت از لحاظ ديمانسيون فاقد بعد هستش. البته اين برداشت شخصي منه. شايد هم در شرايط خاصي قابل تعريف باشه.

موفق باشين.
88/8/20

davy jones
11-11-2009, 13:25
سئال اولم منظورم اون A معادله رو بدست بیارید.

به اين راحتي ها قابل محاسبه نيست. ابتدا بايد به روش نصف كردن يه مقداري رو كه به ريشه معادله نزديكه رو بدست آورد بعد با روش نيوتون تقريب زد. تازه آخرش هم جواب با تقريب خواهد بود و جواب دقيق بدست نمياد.

موفق باشين.
88/8/20

mfani2008
12-11-2009, 13:08
سلام يك سوال داشتم كه خيلي آسونه كسي هست كمك كنه ؟ :D

K مهره روي يك ميز در يك دسته قرار دار ند . قانون بازي اينگونه است كه در هر حركت ، بايد از يك د سته كه بيش از يك مهره دارد، يك مهره دور انداخته شود و سپس يكي از دست هها كه بيش از يك مهره دارد را به دو بخش ناتهي تقسيم كرد . كوچكترين k بزرگتر از ۱۳۸۸ را ب ه نحوي بيابيد كه پس از شروع با k مهره و بعد از چند حركت، بتوان به وضعيتي رسيد كه روي ميز تنها دست ههايي با ۳ مهره وجود داشته باشد؟

mfani2008
12-11-2009, 13:30
K مهره روي يك ميز در يك دسته قرار دارند . قانون بازي اينگونه است كه در هر حركت، بايد از يك دسته كه بيش از يك مهره دارد،يك مهره دور انداخته شود و سپس يكي از دسته ها كه بيش از يك مهره دارد را به دو بخش ناتهي تقسيم كرد . كوچكترين Kبزرگتر از ۱۳۸۸ را به نحوي بيابيد كه پس از شروع با K مهره و بعد از چند حركت، بتوان به وضعيتي رسيد كه روي ميز تنها دسته هايي با ۳ مهره وجود داشته باشد؟

CATALONIA
12-11-2009, 19:24
سلام به همه ی دوستان
من یه چند تا سوال پیش پا افتاده برای شما استادان داشتم که اگه ممکنه کامل کامل راه حلشو بنویسید
باتشکر

 الف) یک مجموعه ی سه عضوی بنویسید که از هر دو عضو ان یکی زیر مجمو عه ی دیگری باشد . 
ب)  یک مجموعه ی سه عضوی بنویسید که هر عضو ان زیر مجمو عه ی ان مجموعه باشد .


1- آیا چند جمله ای n به توان 2 + n + 41 برای تولید اعداد اول به ازای هر مقدار درست است ؟ چرا؟ 
2- عدد 4321 اول است یا مرکب ؟جرا ؟ اگه میشه یه توضیحی بدین کلا که چطور اعدادی که بزرگن رو میشه او یا مرکب بودنشو تشخیص داد 
3- در میان 100 عدد متوالی زیر چند عدد اول وجود دارد؟
101+!101, ..., 3+!101, 2+!101
کدام یک از اعداد زیر اول میباشند کدام مرکب چرا ؟ 
1021 
11111
5 به توان 211 + 5 به توان 212 + 8
1376 به توان 1978 + 1375 به توان 1977 + 1 
2 به توان 100 + 3 به توان 101 + 5 به توان 102 

(25 به توان 11 + 25 به توان 13 +...+25 به توان 99 ) + 24 به توان 52 + 1 
تحقیق کنید عدد 123454321 اول است یا مرکب 
ثابت کنید 1..11 (nرقم ) 6 1...11 (2n رقم) یک عدد مرکب است




ممنون
ببخشید که خیلی زیاد شد 

iranch
12-11-2009, 19:38
سلام
دوستان اگر مبشه کمک کنید
چند تا اشکال دارم
البته نمیدانم به ریاضی چطوری باید تایپ کرد

اثبات کنید
Isinb-sinaI<Ib-aI

b-a) (F'c)=(fb)-(fa))
b-a)xcosc=sinb-sina
Ib-aIxIcoscI=Isinb-sinaI
-1<cosx<1 IcosxI<1
-1<sinx<1 IsinxI<1
Isinb-sinaI=Ib-aIcosc<Ib-aI

I ها منظورم قدر مطلق هست
علامت ها هم کوچکتر و مساوی هست
این یک اثبات کنید از قضیه مقدار میانگین بود
تو خط سوم مشکل دارم
چطوری اخری رو نتیجه داد؟

iranch
12-11-2009, 19:39
b-a) (F'c)=(fb)-(fa))


fb)-(fa=(b-c
بعد از عبارت b-c ضرب در کسر یک c ام می شود عبارت
بلد نبودم کسر بنویسم متاسفانه
بعد
Lnb-Lna=(b-a) بعد عبارت b-a ضرب در کسر یک c ام

اما مشکل اینجاست که بعد شده
Lnb-Lna شده Ln در کسر b تقسیم a

مشکل من اینجاست
که lnb و lna که از هم کم شدند حالا چطوریی سطر بعد Ln bروی a شده؟(Ln بی روی آ شده)
اگر خواص Ln چیزی هست لطفا بگید ممنون
ببخشید




نتیجه( مسئله خودش این رو داده که x بزرگ تر از 0 و کوچک تر از پی دوم است) که زیرش می زنیم

کسینوس x بزرگ تر مساوی از منفی 1 و کوچکتر مساوی از 1 هست
بعد گفته هر عددی بگذاریک که بیشترین ان 1 باشد باز منفی یک میشود
پس f'x<0 است و نزولی است
مشکل اینجاست که یعنی چی هر عددی بگذاریم که بیشترین 1 باشه باز منفی یک خواهد شد
عدد رو کجا و طچوری میزاریم؟



ممنون میشم شرمنده بچه ها
فقط تا اخر امشی وقت دارم لطفا بگید

bn_ha
12-11-2009, 23:35
b-a) (F'c)=(fb)-(fa))


fb)-(fa=(b-c
بعد از عبارت b-c ضرب در کسر یک c ام می شود عبارت
بلد نبودم کسر بنویسم متاسفانه
بعد
Lnb-Lna=(b-a) بعد عبارت b-a ضرب در کسر یک c ام

اما مشکل اینجاست که بعد شده
Lnb-Lna شده Ln در کسر b تقسیم a

مشکل من اینجاست
که lnb و lna که از هم کم شدند حالا چطوریی سطر بعد Ln bروی a شده؟(Ln بی روی آ شده)
اگر خواص Ln چیزی هست لطفا بگید ممنون
ببخشید




نتیجه( مسئله خودش این رو داده که x بزرگ تر از 0 و کوچک تر از پی دوم است) که زیرش می زنیم

کسینوس x بزرگ تر مساوی از منفی 1 و کوچکتر مساوی از 1 هست
بعد گفته هر عددی بگذاریک که بیشترین ان 1 باشد باز منفی یک میشود
پس f'x<0 است و نزولی است
مشکل اینجاست که یعنی چی هر عددی بگذاریم که بیشترین 1 باشه باز منفی یک خواهد شد
عدد رو کجا و طچوری میزاریم؟



ممنون میشم شرمنده بچه ها
فقط تا اخر امشی وقت دارم لطفا بگید
یکی از خواص لگاریتم این هست که تفاضل به تقسیم تبدیل می شه. Ln هم که لگاریتم هست.

mehdi_7070
13-11-2009, 10:35
3- در میان 100 عدد متوالی زیر چند عدد اول وجود دارد؟
101+!101, ..., 3+!101, 2+!101


هیچ کدام از اعداد اول نیستند:46:

به عنوان مثال:


[ برای مشاهده لینک ، لطفا با نام کاربری خود وارد شوید یا ثبت نام کنید ] es%20...%5Ctimes%20101+2=2%281%5Ctimes%203%5Ctimes %204%5Ctimes%20...%5Ctimes%20101+1%29=2q


برای بقیه هم به همین شکل می‌تونید اثبات کنید.

به طور کلی دنباله‌ای که در آن n عدد متوالی غیراول باشند به شکل زیر است:


[ برای مشاهده لینک ، لطفا با نام کاربری خود وارد شوید یا ثبت نام کنید ] 20...%20,%20%28n+1%29%21+%28n+1%29

melody70
13-11-2009, 15:16
سلام
ببخشید کسی بسط تیلور برای tan وcot رو میدونه؟
ممنون میشم

H.R@Wolf
13-11-2009, 19:21
سلام ممنون ميشم اين سوالا رو برام حل كنيد
1- f(x)=x^3+x آنگاه وارون f(x) را بيابيد .
2-اثبات كنيد ميانگين حسابي n عدد مثبت حقيقي كوچكتر مساوي ميانگين هندسي آنهاست.
با تشكر از دوستان

Life24
13-11-2009, 21:53
یکی از خواص لگاریتم این هست که تفاضل به تقسیم تبدیل می شه. Ln هم که لگاریتم هست.
ایول
حل شد
اون مشکل ها باقی هست

CATALONIA
13-11-2009, 23:30
سلام به همه ی دوستان
من یه چند تا سوال پیش پا افتاده برای شما استادان داشتم که اگه ممکنه کامل کامل راه حلشو بنویسید
باتشکر

 الف) یک مجموعه ی سه عضوی بنویسید که از هر دو عضو ان یکی زیر مجمو عه ی دیگری باشد . 
ب)  یک مجموعه ی سه عضوی بنویسید که هر عضو ان زیر مجمو عه ی ان مجموعه باشد .


1- آیا چند جمله ای n به توان 2 + n + 41 برای تولید اعداد اول به ازای هر مقدار درست است ؟ چرا؟ 
2- عدد 4321 اول است یا مرکب ؟جرا ؟ اگه میشه یه توضیحی بدین کلا که چطور اعدادی که بزرگن رو میشه او یا مرکب بودنشو تشخیص داد 
3- در میان 100 عدد متوالی زیر چند عدد اول وجود دارد؟
101+!101, ..., 3+!101, 2+!101
کدام یک از اعداد زیر اول میباشند کدام مرکب چرا ؟ 
1021 
11111
5 به توان 211 + 5 به توان 212 + 8
1376 به توان 1978 + 1375 به توان 1977 + 1 
2 به توان 100 + 3 به توان 101 + 5 به توان 102 

(25 به توان 11 + 25 به توان 13 +...+25 به توان 99 ) + 24 به توان 52 + 1 
تحقیق کنید عدد 123454321 اول است یا مرکب 
ثابت کنید 1..11 (nرقم ) 6 1...11 (2n رقم) یک عدد مرکب است




ممنون
ببخشید که خیلی زیاد شد 


کسی نیست اینا رو حل کنه !!!!!!!!!!!!!!!!!!؟!!!!!!!!!!!!!!!!!!

میدونم خیلی زیاده
پس اگه ممکنه اینارو حل کنین

الف) یک مجموعه ی سه عضوی بنویسید که از هر دو عضو ان یکی زیر مجمو عه ی دیگری باشد . 
ب)  یک مجموعه ی سه عضوی بنویسید که هر عضو ان زیر مجمو عه ی ان مجموعه باشد .


1- آیا چند جمله ای n به توان 2 + n + 41 برای تولید اعداد اول به ازای هر مقدار درست است ؟ چرا؟ 
2- عدد 4321 اول است یا مرکب ؟جرا ؟ اگه میشه یه توضیحی بدین کلا که چطور اعدادی که بزرگن رو میشه او یا مرکب بودنشو تشخیص داد
3-ثابت کنید 1..11 (nرقم ) 6 1...11 (2n رقم) یک عدد مرکب است

eh_mn
14-11-2009, 12:55
کسی نیست اینا رو حل کنه !!!!!!!!!!!!!!!!!!؟!!!!!!!!!!!!!!!!!!

میدونم خیلی زیاده
پس اگه ممکنه اینارو حل کنین

الف) یک مجموعه ی سه عضوی بنویسید که از هر دو عضو ان یکی زیر مجمو عه ی دیگری باشد . 
ب)  یک مجموعه ی سه عضوی بنویسید که هر عضو ان زیر مجمو عه ی ان مجموعه باشد .


1- آیا چند جمله ای n به توان 2 + n + 41 برای تولید اعداد اول به ازای هر مقدار درست است ؟ چرا؟ 
2- عدد 4321 اول است یا مرکب ؟جرا ؟ اگه میشه یه توضیحی بدین کلا که چطور اعدادی که بزرگن رو میشه او یا مرکب بودنشو تشخیص داد
3-ثابت کنید 1..11 (nرقم ) 6 1...11 (2n رقم) یک عدد مرکب است


لطفا سری به اتاق « انتشار درست عبارات و فرمول های ریاضی در سایت » بزنید. ممنون!


(الف) [ برای مشاهده لینک ، لطفا با نام کاربری خود وارد شوید یا ثبت نام کنید ]{&space;\{1,2&space;\},&space;\{&space;1\},&space;\O&space;\}،
(ب) [ برای مشاهده لینک ، لطفا با نام کاربری خود وارد شوید یا ثبت نام کنید ]{&space;\{&space;\{\O\},\O&space;\},\{\O\},\O&space;\}

1- داریم [ برای مشاهده لینک ، لطفا با نام کاربری خود وارد شوید یا ثبت نام کنید ]^2&plus;n&plus;41=n(n&plus;1)&plus;41، حالا n رو چی انتخاب کنیم تا این عدد حتما مرکب بشه؟

2- یک قضیه‌ی معروف: اگر n یک عدد طبیعی مرکب و p عامل اول آن باشد آنگاه [ برای مشاهده لینک ، لطفا با نام کاربری خود وارد شوید یا ثبت نام کنید ]{n}. شاید بشه از این قضیه استفاده کرد!

3- قوانین معروف مربوط به بخش‌پذیری بر اعداد 2 و 3 و 5 و 4 و 6 و ... را بررسی کنید!

موفق باشید

davy jones
14-11-2009, 15:37
سلام
ببخشید کسی بسط تیلور برای tan وcot رو میدونه؟
ممنون میشم

طبق تعریف خودت میتونی بشینی و تا هر جمله که دلت خواست حساب کنی. فکر نکنم رابطه ای بین ضرایب بسط تیلور تانژانت وجود داشته باشه به جز اینکه ضرایب توانهای زوج صفره.

موفق باشین.

davy jones
14-11-2009, 15:40
ثابت کنید 1..11 (nرقم ) 6 1...11 (2n رقم) یک عدد مرکب است

مجموع ارقام این عدد برابر با 3n+6 میشه که واضحه بر 3 بخشپذیره. پس مرکبه.

موفق باشین.

eh_mn
14-11-2009, 19:43
سلام
ببخشید کسی بسط تیلور برای tan وcot رو میدونه؟
ممنون میشم

سلام

كتابي با عنوان «Mathematical Handbook of Formulas and Tables» توسط « Murray Spiegel » و سايرين، نوشته شده است كه در آن جدول‌ها و فرمول‌هاي رياضي مورد نياز براي رشته‌هاي آمار، مهندسي و مديريت مالي و... وجود دارد.

نسخه‌ي الكترونيكي اين كتاب را مي‌توانيد از

برای مشاهده محتوا ، لطفا وارد شوید یا ثبت نام کنید
دانلود كنيد.

ويرايش قديمي‌تر اين كتاب توسط «علي و محمد حسيني» با عنوان «مجموعه فرمول‌ها و جدول‌هاي رياضي»ترجمه شده است.

بسط تيلور tan و cot به صورت زير است

[ برای مشاهده لینک ، لطفا با نام کاربری خود وارد شوید یا ثبت نام کنید ]{n=1}^\infty&space;\frac{2^{2n}(2^ {2n}-1)B_n}{(2n)!}x^{2n-1}
و

[ برای مشاهده لینک ، لطفا با نام کاربری خود وارد شوید یا ثبت نام کنید ]{1}{x}-\sum_{n=1}^\infty&space;\frac{2^{2n}B_n}{(2n)!}x^{2n-1}
كه در آن Bnها كه به اعداد برنولي معروفند ضرايب [ برای مشاهده لینک ، لطفا با نام کاربری خود وارد شوید یا ثبت نام کنید ]{x^n}/{n!} در بسط تابع [ برای مشاهده لینک ، لطفا با نام کاربری خود وارد شوید یا ثبت نام کنید ]{x}{e^x-1} هستند يعني


[ برای مشاهده لینک ، لطفا با نام کاربری خود وارد شوید یا ثبت نام کنید ]{x}{e^x-1}=\sum_{n=0}^\infty&space;B_n\frac{x^n}{n!}

همونطور كه davy jones گفتن، گويا فرمول صريح نداره و شايد اگر تا همون جمله‌اي كه لازم دارين حساب كنين راحت تره باشه.


‌‌

H.R@Wolf
15-11-2009, 15:02
سلام ممنون ميشم اين سوالا رو برام حل كنيد
1- f(x)=x^3+x آنگاه وارون f(x) را بيابيد .
2-اثبات كنيد ميانگين حسابي n عدد مثبت حقيقي كوچكتر مساوي ميانگين هندسي آنهاست.
با تشكر از دوستان
كسي نيست جواب بده؟؟!!:19::19::13::13:

CATALONIA
15-11-2009, 16:13
کسی نیست اینا رو حل کنه !!!!!!!!!!!!!!!!!!؟!!!!!!!!!!!!!!!!!!


میدونم خیلی زیاده 

پس اگه ممکنه اینارو حل کنین

الف) یک مجموعه ی سه عضوی بنویسید که از هر دو عضو ان یکی زیر مجمو عه ی دیگری باشد . 
ب)  یک مجموعه ی سه عضوی بنویسید که هر عضو ان زیر مجمو عه ی ان مجموعه باشد .


1- آیا چند جمله ای n به توان 2 + n + 41 برای تولید اعداد اول به ازای هر مقدار درست است ؟ چرا؟ 
2- عدد 4321 اول است یا مرکب ؟جرا ؟ اگه میشه یه توضیحی بدین کلا که چطور اعدادی که بزرگن رو میشه او یا مرکب بودنشو تشخیص داد
3-ثابت کنید 1..11 (nرقم ) 6 1...11 (2n رقم) یک عدد مرکب است


اصلاح شده ی سوالات 
فکر نمیکنم که دیگه مشکلی باشه اگه بود بگین تا توضیح بدم 


با تشکر از دوست عزیزی که پیشنهاد کرد سوالات را تصحیح کنم 
  اگه میشه  این قسمت رو توضیح بدین
اگه میشه یه توضیحی بدین کلا که چطور اعدادی که بزرگن رو میشه او یا مرکب بودنشو تشخیص داد
غیر از روش تجزیه 





الف) یک مجموعه ی سه عضوی بنویسید که از هر دو عضو ان یکی زیر مجمو عه ی دیگری باشد . 
ب)  یک مجموعه ی سه عضوی بنویسید که هر عضو ان زیر مجمو عه ی ان مجموعه باشد .


  1- آیا چند جمله ای [ برای مشاهده لینک ، لطفا با نام کاربری خود وارد شوید یا ثبت نام کنید ]^2&plus;n&plus;41 برای تولید اعداد اول به ازای هر مقدار درست است ؟ چرا؟ 
2- عدد 4321 اول است یا مرکب ؟جرا ؟ اگه میشه یه توضیحی بدین کلا که چطور اعدادی که بزرگن رو میشه او یا مرکب بودنشو تشخیص داد
  3-ثابت کنید 1..11 6 1...11  یک عدد مرکب است

                     (nرقم ) (2n رقم) 

mass0ood
18-11-2009, 01:18
سلام خدمت دوستان.
می خواستم بپرسم که دلتای یه معادله رو با هر درجه ای چه جوری به دست میارن؟
مثلا همه می دونیم که دلتای معادله ی درجه 2 می شه b^2 - 4ac
در ضمن اگه ممکنه معادل انگلیسی دلتا رو هم بنویسید

با تشکر

bn_ha
22-11-2009, 23:15
سلام
یک فرمول کلی برای حل چنین انتگرالهایی بیابید
[ برای مشاهده لینک ، لطفا با نام کاربری خود وارد شوید یا ثبت نام کنید ](1&plus;x^{m})^{n}x^{p}dx

bn_ha
23-11-2009, 13:12
ببخشید کسی نیست این رو حل کنه؟چون خیلی برام مهمه

bn_ha
23-11-2009, 18:31
مسئله حل شد.
ممنون

afshin b
23-11-2009, 20:51
مسئله حل شد.
ممنون

ميشه جوابشو بزاري؟

Desmond
23-11-2009, 20:55
آره اگه جوابشو بزاری ممنون می شم.

Desmond
23-11-2009, 20:57
سلام خدمت دوستان.
می خواستم بپرسم که دلتای یه معادله رو با هر درجه ای چه جوری به دست میارن؟
مثلا همه می دونیم که دلتای معادله ی درجه 2 می شه b^2 - 4ac
در ضمن اگه ممکنه معادل انگلیسی دلتا رو هم بنویسید

با تشکر
دلتا فقط برای معادله درجه 2 هست و درجه های بالاتر روش های خیلی پیشرفتی هست تا آنجا که یه دکتر ریاضی هم نمی تونه حلش کنه.

hamidehsadat
23-11-2009, 21:23
من دانشجوی رشته کارشناسی کامپیوتر هستم می خواستم ببینم از دوستان کسی میتونه به من در ریاضیات گسسته کمک کنه؟

bn_ha
24-11-2009, 00:35
حلش این طوریه:
1)اگر P صحیح و مثبت باشد بسط می دهیم
2)اگر P منفی باشد باید عبارت داخل پرانتز رو مساوی Z به توان r بگیریم که r ک م م مخرجهای n و m است
3)اگر m+1 /n (کلا m+1 تقسیم بر n) عدد صحیح شد باید عبارت داخل پرانتز رو مساوی Z به توان r بگیریم که در آن r مخرج P است.

bn_ha
24-11-2009, 00:39
مثال:
[ برای مشاهده لینک ، لطفا با نام کاربری خود وارد شوید یا ثبت نام کنید ]{dx}{\sqrt{x^{3}}}\sqrt[-3]{1&plus;\sqrt[4]{x^{3}}}

bn_ha
24-11-2009, 00:42
حال کسی هست این انتگرال رو برای من حل کنه؟
ممنون
[ برای مشاهده لینک ، لطفا با نام کاربری خود وارد شوید یا ثبت نام کنید ]{\0&space;}^{\infty&space;}e^{-x^{2}}dx

bn_ha
24-11-2009, 19:31
مثل این که این سوال هم بی جواب موند

eh_mn
25-11-2009, 17:59
حال کسی هست این انتگرال رو برای من حل کنه؟
ممنون
[ برای مشاهده لینک ، لطفا با نام کاربری خود وارد شوید یا ثبت نام کنید ]{\0&space;}^{\infty&space;}e^{-x^{2}}dx

گويا در اينجا حل شده

برای مشاهده محتوا ، لطفا وارد شوید یا ثبت نام کنید

! Time
26-11-2009, 15:50
سلام،
می خواستم بدانم که آیا sin و tan برداری هستند یا نه؟ همراه با دلیل آن؟
متشکرم.

آقا کسی دگیه ای نیست به ما جواب بده؟ :13:

CATALONIA
27-11-2009, 10:41
سلام

4 گاو سیاه و 3 گاو سفید در مدت 5 روز شیر ان ها برابر است با 5 گاو سفید و 3 گاو سیاه در مدت 4 روز شیر 8 گاو سفید با چند گاو سیاه برابر است ؟؟

attractive_girl
28-11-2009, 01:08
سلام ، جواب این دو سوال رو میخواستم(البته برای خودم نمیخوام)
تا جایی که میشه راه حلش ساده باشه ،ممنون
1-

[ برای مشاهده لینک ، لطفا با نام کاربری خود وارد شوید یا ثبت نام کنید ]

2-

[ برای مشاهده لینک ، لطفا با نام کاربری خود وارد شوید یا ثبت نام کنید ]

darklord_8
29-11-2009, 15:53
با سلام و خسته نباشید
یه سوال داشتم
توی تابع دو ضابطه ای زیر اگر از نظر هندسی به نمودار نگاه کنیم هرچی از سمت چپ به صفر نزدیک بشیم مشتق باید به صفر نزدیکتر بشه (طوری که اگر تابع تو صفر تعریف شده بود باید برابر صفر بشه)
ولی اگر برای صفر منفی از تعریف مشتق استفاده کنیم جوابی که به ما میده مثبت بینهایته.
ایراد کجاست؟
ممنون

[ برای مشاهده لینک ، لطفا با نام کاربری خود وارد شوید یا ثبت نام کنید ]

mehdi_7070
29-11-2009, 18:14
با سلام و خسته نباشید
یه سوال داشتم
توی تابع دو ضابطه ای زیر اگر از نظر هندسی به نمودار نگاه کنیم هرچی از سمت چپ به صفر نزدیک بشیم مشتق باید به صفر نزدیکتر بشه (طوری که اگر تابع تو صفر تعریف شده بود باید برابر صفر بشه)
ولی اگر برای صفر منفی از تعریف مشتق استفاده کنیم جوابی که به ما میده مثبت بینهایته.
ایراد کجاست؟
ممنون

[ برای مشاهده لینک ، لطفا با نام کاربری خود وارد شوید یا ثبت نام کنید ]

مشتق یعنی خط مماس. خط مماس در نقطه صفر منفی عمود هست. به خاطر همین بی‌نهایت شده.

amintnt
30-11-2009, 09:52
با سلام و خسته نباشید
یه سوال داشتم
توی تابع دو ضابطه ای زیر اگر از نظر هندسی به نمودار نگاه کنیم هرچی از سمت چپ به صفر نزدیک بشیم مشتق باید به صفر نزدیکتر بشه (طوری که اگر تابع تو صفر تعریف شده بود باید برابر صفر بشه)
ولی اگر برای صفر منفی از تعریف مشتق استفاده کنیم جوابی که به ما میده مثبت بینهایته.
ایراد کجاست؟
ممنون

[ برای مشاهده لینک ، لطفا با نام کاربری خود وارد شوید یا ثبت نام کنید ]
تعريف مشتق رو براي تابع مينويسيم:
صورت: (منفي x^2) منهاي (صفر)
مخرج: x منهاي صفر

جواب ميشه منفي x

که اگه صفر رو جاگذاري کنيم جواب ميشه صفر...

چرا بينهايت؟



سلام ، جواب این دو سوال رو میخواستم(البته برای خودم نمیخوام)
تا جایی که میشه راه حلش ساده باشه ،ممنون
1-

[ برای مشاهده لینک ، لطفا با نام کاربری خود وارد شوید یا ثبت نام کنید ]اندازه این تصویر کوچک شده است ! برای مشاهده تصویر اصلی اینجا کلیک کنید . اندازه اصلی تصویر 783x501 و حجم آن 27 کیلوبایت میباشد.[ برای مشاهده لینک ، لطفا با نام کاربری خود وارد شوید یا ثبت نام کنید ]

AB+BC=AC

AC رو به دست مياريم... حالا نقطه ي ابتداي بردار که A هست داريم ، و ميدونيم انتهاي بردار منهاي ابتداي بردار ميشه خود بردار... خوب بردار رو که داريم، نقطه ي ابتدا هم همونطور که گفتم داريم.... حالا نقطه ي انتهاي بردار که همون C هست به دست مياريم... و در آخر C+A تقسيم بر 2 ميشه نقطه ي D... و C-D ميشه جواب...



[ برای مشاهده لینک ، لطفا با نام کاربری خود وارد شوید یا ثبت نام کنید ]
zx+xA+Ac=zc
zy+yB+Bc=zc
و با توجه به فرض مسئله:
Ac=-BC
zx=-zy

دو معادله ي اول رو جمع ميکنيم ميرسيم به xA+yB=2zc ... که طرفين رو تقسيم بر دو ميکنيم و ميرسيم به جواب...

darklord_8
30-11-2009, 10:59
مشتق یعنی خط مماس. خط مماس در نقطه صفر منفی عمود هست. به خاطر همین بی‌نهایت شده.
درسته . ولی ما هنوز به نقطه صفر نرسیدیم. اگر روی صفر منظور بود تعریف مشتق شیب خط بین دو نقطه انفصال رو میداد که درست هم هست و مثبت بینهایت میشه. ولی از روی نمودار که نگاه میکنم به نظرم میاد شیب باید به صفر نزدیک باشه

صورت: (منفي x^2) منهاي (صفر)
منهای صفر یا یک؟

davy jones
30-11-2009, 12:30
سلام

4 گاو سیاه و 3 گاو سفید در مدت 5 روز شیر ان ها برابر است با 5 گاو سفید و 3 گاو سیاه در مدت 4 روز شیر 8 گاو سفید با چند گاو سیاه برابر است ؟؟

گاو سیاه = x
گاو سفید = y
(4x+3y)/5=(3x+5y)/4
16x+12y=15x+25y
x=13y
8y=8/13x

موفق باشین.
88/9/9

afshin b
30-11-2009, 13:27
گاو سیاه = x
گاو سفید = y
(4x+3y)/5=(3x+5y)/4
16x+12y=15x+25y
x=7y
8y=8/7x=x+1/7x

شیر 8 گاو سفید با یک گاو سیاه به علاوه ی یک هفتم یک گاو سیاه برابر است.

موفق باشین.
88/9/9

دوست عزيز تو ساده كردن اشتباه كردي
x=13y
8y=(8/13)x

davy jones
30-11-2009, 14:35
دوست عزيز تو ساده كردن اشتباه كردي
x=13y
8y=(8/13)x

از تذکرتون ممنونم. حق با شماست.

پست قبلی ویرایش شد.

amintnt
30-11-2009, 14:38
منهای صفر یا یک؟
بله درسته... بايد ضابطه ي بالايي در نظر گرفته ميشد... عذر ميخوام...


توی تابع دو ضابطه ای زیر اگر از نظر هندسی به نمودار نگاه کنیم هرچی از سمت چپ به صفر نزدیک بشیم مشتق باید به صفر نزدیکتر بشه (طوری که اگر تابع تو صفر تعریف شده بود باید برابر صفر بشه)

به سوال که فکر کردم خودمم گيج شدم يه خورده.... توجيهي که به ذهنم اومد اين بود که... شما ميگي به صفر نزديک بشيم... اما چقدر؟ ما توي مشتقي که از اين تابع ميگيريم به خود صفر هم ميرسيم... توي تعريف مشتق داشتيم که اگه تابع روي بازه ي (بسته) [a,b] پيوسته باشه... اونوقت روي بازه ي (باز) (a,b) مشتق پذيره... در اين مثال شرط پيوستگي رعايت نشده که بخوايم مشتق بگيريم... در واقع بازه ي مشتق گيري شما هست از منفي اپسيلون بسته تا صفر بسته... که واضحه پيوسته نيست...

اميدوارم درست باشه!

:)

darklord_8
01-12-2009, 16:03
به سوال که فکر کردم خودمم گيج شدم يه خورده
خب منم به همین خاطر مطرح کردم. در اصل این سوال رو استاد ریاضی 1 مطرح کرد گفت "خودمم نمیدونم چرا اینطوریه" :18: هرکی بگه یه نمره میگیره!
توضیحات شما هم درسته ولی طبق فرضی که استاد گفت درسته که پیوستگی لازمه ولی اینجا اگه ازش چشم پوشی کنیم (ما پیوستگی رو برای ضوابطی بررسی میکنیم که از پیوستگیش مطمءن نیستیم. ولی اینجا تو نمودار پیوستگی واضحه)
حالا میمونه اینکه این اپسیلون چقدره که باعث میشه شیب بینهایت بشه؟ به نظر من تا وقتی ما به صفر نرسیدیم نباید به ما بینهایت بده{مگر دقیقا تو صفر} . غیر از این باید یه عدد خیلی ریز نزدیک به صفر بده.
همین باعث این شده که خودمم گیج بشم :41:

bn_ha
02-12-2009, 01:21
آقا تو روخدا این سول رو تا امروز ظهر برام حل کنید ممنون
[ برای مشاهده لینک ، لطفا با نام کاربری خود وارد شوید یا ثبت نام کنید ]{a}^{c}f(x)dx=k\int_{a}^{b}f(x)dx
اثبات کنید عددی مانند c بین a و b وجود دارد که در رابطه ی بالا صدق کند. k بین 1 و 0 است.

bn_ha
02-12-2009, 12:20
کسی نیست جواب بده؟
ممنون

mahdishojaee
02-12-2009, 12:20
سلام . کسی میتونه لطفا این معادله ی دیفرانسیل رو حل کنه ؟
y'=(x+y)/xy
خودم فکر می کنم باید براش فاکتور انتگرال ساز پیدا کرد و از اون راه هم نتونستم حل کنم.

mahsa1469
02-12-2009, 15:28
چند تا سوال داشتم تا شب جوابشون رو می خوام:
اولیش انتگرال زیره:
[ برای مشاهده لینک ، لطفا با نام کاربری خود وارد شوید یا ثبت نام کنید ] \frac{e^{3x}}{\sqrt{5+e^{x}}}dx

دومیش هم اینه که عبارت زیر را ثابت کنید:

[ برای مشاهده لینک ، لطفا با نام کاربری خود وارد شوید یا ثبت نام کنید ]^{x}\geq x+1

اگه اشتباه نکنم به ازای هر xبود و با استفاده از اون دو عبارت زیر را ثابت کنید

برای[ برای مشاهده لینک ، لطفا با نام کاربری خود وارد شوید یا ثبت نام کنید ] x<1

[ برای مشاهده لینک ، لطفا با نام کاربری خود وارد شوید یا ثبت نام کنید ]^{x}\leq \frac{1}{x-1}

و
[ برای مشاهده لینک ، لطفا با نام کاربری خود وارد شوید یا ثبت نام کنید ](\frac{x+1}{x})\leq \frac{1}{x}\leq ln(\frac{x}{x-1})

و آخریش هم حد زیره::

[ برای مشاهده لینک ، لطفا با نام کاربری خود وارد شوید یا ثبت نام کنید ]{x\to 0^{+}}(1-xlnx)^{\frac{1}{x}}

سوالا از ریاضی 1 و مباحث میان ترمه

bn_ha
02-12-2009, 18:43
[ برای مشاهده لینک ، لطفا با نام کاربری خود وارد شوید یا ثبت نام کنید ](x)\geq&space;f(0)
چند تا سوال داشتم تا شب جوابشون رو می خوام:
اولیش انتگرال زیره:
[ برای مشاهده لینک ، لطفا با نام کاربری خود وارد شوید یا ثبت نام کنید ] \frac{e^{3x}}{\sqrt{5+e^{x}}}dx

دومیش هم اینه که عبارت زیر را ثابت کنید:

[ برای مشاهده لینک ، لطفا با نام کاربری خود وارد شوید یا ثبت نام کنید ]^{x}\geq x+1

اگه اشتباه نکنم به ازای هر xبود و با استفاده از اون دو عبارت زیر را ثابت کنید

برای[ برای مشاهده لینک ، لطفا با نام کاربری خود وارد شوید یا ثبت نام کنید ] x<1

[ برای مشاهده لینک ، لطفا با نام کاربری خود وارد شوید یا ثبت نام کنید ]^{x}\leq \frac{1}{x-1}

و
[ برای مشاهده لینک ، لطفا با نام کاربری خود وارد شوید یا ثبت نام کنید ](\frac{x+1}{x})\leq \frac{1}{x}\leq ln(\frac{x}{x-1})

و آخریش هم حد زیره::

[ برای مشاهده لینک ، لطفا با نام کاربری خود وارد شوید یا ثبت نام کنید ]{x\to 0^{+}}(1-xlnx)^{\frac{1}{x}}

سوالا از ریاضی 1 و مباحث میان ترمه
سلام
اینجا مثل اینکه کسی به کسی جواب نمی ده
[ برای مشاهده لینک ، لطفا با نام کاربری خود وارد شوید یا ثبت نام کنید ]{e^{3x}}{\sqrt{5&plus;e^{x}}}dx=\in t&space;\frac{e^{2x}\times&space;e^{x}dx&space;}{\sqrt{5&plus;e^{x}}}=\in t&space;\frac{(u-5)^{2}du}{\sqrt{u}}
در اینجا از تغییر متغیر[ برای مشاهده لینک ، لطفا با نام کاربری خود وارد شوید یا ثبت نام کنید ]^{x}=uاستفاده کردم.که [ برای مشاهده لینک ، لطفا با نام کاربری خود وارد شوید یا ثبت نام کنید ]^{x}dx=du بقیه اش هم که پرانتز رو باز می کنیم و بر مخرج تقسیم کرده و خیلی راحت حله

2)تابع جدیدی رو تعریف می کنیم:
[ برای مشاهده لینک ، لطفا با نام کاربری خود وارد شوید یا ثبت نام کنید ](x)=e^{x}-x-1
[ برای مشاهده لینک ، لطفا با نام کاربری خود وارد شوید یا ثبت نام کنید ]{}'(x)=e^{x}-1
این تابع به ازای x های بزرگتر مساوی صفر صعودی و در نتیجه:
[ برای مشاهده لینک ، لطفا با نام کاربری خود وارد شوید یا ثبت نام کنید ](x)\geq&space;f(0)
که همون حکم مسئله هست
و به ازای x های کمتر از صفر نزولی پس:
[ برای مشاهده لینک ، لطفا با نام کاربری خود وارد شوید یا ثبت نام کنید ](x)>&space;f(0)

برای دومی تابع را به صورت زیر تعریف می کنیم و مثل بالا انجام می دیم
[ برای مشاهده لینک ، لطفا با نام کاربری خود وارد شوید یا ثبت نام کنید ]^{x}(x-1)-1
آیا فرض
[ برای مشاهده لینک ، لطفا با نام کاربری خود وارد شوید یا ثبت نام کنید ] x<1
برای دومی هم هست یا نه؟

eh_mn
03-12-2009, 00:27
آقا تو روخدا این سول رو تا امروز ظهر برام حل کنید ممنون
[ برای مشاهده لینک ، لطفا با نام کاربری خود وارد شوید یا ثبت نام کنید ]{a}^{c}f(x)dx=k\int_{a}^{b}f(x)dx
اثبات کنید عددی مانند c بین a و b وجود دارد که در رابطه ی بالا صدق کند. k بین 1 و 0 است.

قرار دهيد


[ برای مشاهده لینک ، لطفا با نام کاربری خود وارد شوید یا ثبت نام کنید ](x)=\int_a^xf(t)dt

بنا به قضيه‌اي g تابعي پيوسته است. از طرفي چون k بين صفر و يك است پس

[ برای مشاهده لینک ، لطفا با نام کاربری خود وارد شوید یا ثبت نام کنید ](a)=0<\underbrace{k\int_a^bf(x)dx}\limits_{t}<\int_a^bf(x)dx=g(b)

از پيوستگي g (اين كه g در خاصيت مقدار مياني صدق مي‌كند) و اين نامساوي چطور مي‌توان حكم را نتيجه گرفت؟

bn_ha
03-12-2009, 14:22
قرار دهيد


[ برای مشاهده لینک ، لطفا با نام کاربری خود وارد شوید یا ثبت نام کنید ](x)=\int_a^xf(t)dt

بنا به قضيه‌اي g تابعي پيوسته است. از طرفي چون k بين صفر و يك است پس

[ برای مشاهده لینک ، لطفا با نام کاربری خود وارد شوید یا ثبت نام کنید ](a)=0<\underbrace{k\int_a^bf(x)dx}\limits_{t}<\int_a^bf(x)dx=g(b)

از پيوستگي g (اين كه g در خاصيت مقدار مياني صدق مي‌كند) و اين نامساوي چطور مي‌توان حكم را نتيجه گرفت؟
خوب اگه مي دونستم كه نمي پرسيدم

eh_mn
03-12-2009, 20:36
خوب اگه مي دونستم كه نمي پرسيدم

خاصيت مقدار مياني براي تابع g ميگه كه اگر [ برای مشاهده لینک ، لطفا با نام کاربری خود وارد شوید یا ثبت نام کنید ](a)<t<g(b) آنگاه c بين a و b وجود داره كه g(c)=t. با توجه به انتخاب t ، اين يعني همون حكم مسأله.

اگه واضح نيست بفرماييد تا بيشتر در موردش صحبت كنيم.

bnmnb
04-12-2009, 01:45
خاصيت مقدار مياني براي تابع g ميگه كه اگر [ برای مشاهده لینک ، لطفا با نام کاربری خود وارد شوید یا ثبت نام کنید ](a)<t<g(b) آنگاه c بين a و b وجود داره كه g(c)=t. با توجه به انتخاب t ، اين يعني همون حكم مسأله.

اگه واضح نيست بفرماييد تا بيشتر در موردش صحبت كنيم.
واقعا ممنونم.:40:كاملا فهميدم

bnmnb
04-12-2009, 02:46
من واقعا شرمنده ام كه اينقدر سوال مي پرسم و از كساني كه كمكم مي كنن تشكر فراوان مي كنم
3 تا سوال دارم
1)مقدار حد زير را با تبديل به انتگرال معين بيابيد(n به سمت بي نهايت ميل مي كند)
[ برای مشاهده لینک ، لطفا با نام کاربری خود وارد شوید یا ثبت نام کنید ](1&plus;\frac{1}{n})(1&plus;\frac{2}{n})^{\frac {1}{2}}(1&plus;\frac{3}{n})^{\frac{1}{3}}...(1&plus;\frac{n} {n})^{\frac{1}{n}}
(بين پرانتز ها ضرب هست)

2)مقدار انتگرال زير را بيابيد:
[ برای مشاهده لینک ، لطفا با نام کاربری خود وارد شوید یا ثبت نام کنید ]{0}^{2\pi&space;}\frac{dx}{3&plus;cos&space;x}

3)يك رابطه ي بازگشتي براي انتگرال زير بيابيد:
[ برای مشاهده لینک ، لطفا با نام کاربری خود وارد شوید یا ثبت نام کنید ]{dx}{(1&plus;x^{2})^{n}}

ببخشيد كلا روش حل مسائلي مثل سوال 3 چه طوريه؟ممنون

afshin b
04-12-2009, 12:24
چند تا سوال داشتم تا شب جوابشون رو می خوام:
اولیش انتگرال زیره:
[ برای مشاهده لینک ، لطفا با نام کاربری خود وارد شوید یا ثبت نام کنید ] \frac{e^{3x}}{\sqrt{5+e^{x}}}dx

دومیش هم اینه که عبارت زیر را ثابت کنید:

[ برای مشاهده لینک ، لطفا با نام کاربری خود وارد شوید یا ثبت نام کنید ]^{x}\geq x+1

اگه اشتباه نکنم به ازای هر xبود و با استفاده از اون دو عبارت زیر را ثابت کنید

برای[ برای مشاهده لینک ، لطفا با نام کاربری خود وارد شوید یا ثبت نام کنید ] x<1

[ برای مشاهده لینک ، لطفا با نام کاربری خود وارد شوید یا ثبت نام کنید ]^{x}\leq \frac{1}{x-1}

و
[ برای مشاهده لینک ، لطفا با نام کاربری خود وارد شوید یا ثبت نام کنید ](\frac{x+1}{x})\leq \frac{1}{x}\leq ln(\frac{x}{x-1})

و آخریش هم حد زیره::

[ برای مشاهده لینک ، لطفا با نام کاربری خود وارد شوید یا ثبت نام کنید ]{x\to 0^{+}}(1-xlnx)^{\frac{1}{x}}

سوالا از ریاضی 1 و مباحث میان ترمه


در مورد اون حد آخري: حدو برابر y فرض ميكني، از دو طرف Ln ميگيري و با كمك هوپيتال حلش ميكني، كه Lny=0 بدست مياد يعني y=1

mohamad_reza72
04-12-2009, 16:59
کوتاهترین فاصله (3و1)به نمودار x2+1


قرمزه توان x

bnmnb
04-12-2009, 18:37
کوتاهترین فاصله (3و1)به نمودار x2+1


قرمزه توان x
فكر مي كنم مشكلتون توي حل معادله مشتق باشه.شما براي يافتن اون بايد از راه مقدار مياني بريد و بازه رو كم و كمتر كنيد تا به جواب برسيد(چيزي كه به ذهن من رسيد):10:

bioelec_352
05-12-2009, 09:56
سلام دوستان
۱ سوال:
یادم می اد ۱ سایتی که online انتگرال می گرفت رو یه جایی دیدم (شاید توی همین سایت بود)ولی الان هرچی دنبالش می گردم بیداش نمی کنم؟؟؟؟ میشه ۱ی کمکم کنه....
با تشکر

bnmnb
05-12-2009, 12:53
سلام دوستان
۱ سوال:
یادم می اد ۱ سایتی که online انتگرال می گرفت رو یه جایی دیدم (شاید توی همین سایت بود)ولی الان هرچی دنبالش می گردم بیداش نمی کنم؟؟؟؟ میشه ۱ی کمکم کنه....
با تشکر

برای مشاهده محتوا ، لطفا وارد شوید یا ثبت نام کنید:10:

Life24
05-12-2009, 12:55
میگم این زاوایه ها رو چطور حساب باید کرد
تتا برابر هست با تانژانت 270 روی 60 مثلا
این طوری

eh_mn
05-12-2009, 13:04
سلام دوستان
۱ سوال:
یادم می اد ۱ سایتی که online انتگرال می گرفت رو یه جایی دیدم (شاید توی همین سایت بود)ولی الان هرچی دنبالش می گردم بیداش نمی کنم؟؟؟؟ میشه ۱ی کمکم کنه....
با تشکر

سلام

عبارت Integration Online و Calculate Integrals Online رو با استفاده از گوكل جستجو كردم.

محاسبه‌ي انتگرال با به صورت برخط، توسط نرم‌افزار متمتيكا :

برای مشاهده محتوا ، لطفا وارد شوید یا ثبت نام کنید 


برای مشاهده محتوا ، لطفا وارد شوید یا ثبت نام کنید

Life24
06-12-2009, 13:09
میگم این زاوایه ها رو چطور حساب باید کرد
تتا برابر هست با تانژانت 270 روی 60 مثلا
این طوری
تو مسئله های فیزیک حساب کردن این طور زوایه ها رو میدن یا باید حساب کنیم؟
مثلا 140 روی 69

davy jones
07-12-2009, 12:55
کوتاهترین فاصله (3و1)به نمودار x2+1


قرمزه توان x

يه نقطه كلي روي تابع مورد نظر شما به صورت (x,x^2+1) ميشه نشون داد. حالا طبق فيثاغورس فاصله اين نقطه تا نقطه (1,3) به صورت زيره:
d^2=(x-1)^2+(x^2+1-3)^2
كه ما ميخواهيم عبارت بالا مينيمم شود. پس ازش مشتق ميگيريم و ريشه هاي مشتق رو محاسبه ميكنيم.
2x-2+4x^3-8x=0
2x^3-3x-1=0
(x+1)(2x^2-2x-1)=0
كه سه ريشه مد نظر ما عبارت است از منفي يك، (0.5 مثبت و منفي (راديكال 3)/2) كه اگه اين ريشه ها رو مجددا در رابطه فيثاغورس بذاري متوجه ميشي كه يكي از اينا كمترين فاصله رو ميده و اون هم ريشه مربوط به نيم به علاوه يك دوم راديكال 3 هستش.

فكر كنم بقيه اش رو خودت بلدي.

موفق باشين.
88/9/16

davy jones
07-12-2009, 13:04
سلام . کسی میتونه لطفا این معادله ی دیفرانسیل رو حل کنه ؟
y'=(x+y)/xy
خودم فکر می کنم باید براش فاکتور انتگرال ساز پیدا کرد و از اون راه هم نتونستم حل کنم.

y'=(x+y)/xy
y'=1/x+1/y
yy'-1=1/x
ydy=(x+1)dx/x
y^2/2=x+lnx
y=(2x+2lnx)^0.5

موفق باشين.
88/9/16

bnmnb
07-12-2009, 14:17
مقدار حد زير را با تبديل به انتگرال معين بيابيد(n به سمت بي نهايت ميل مي كند)
[ برای مشاهده لینک ، لطفا با نام کاربری خود وارد شوید یا ثبت نام کنید ](1&plus;\frac{1}{n})(1&plus;\frac{2}{n})^{\frac {1}{2}}(1&plus;\frac{3}{n})^{\frac{1}{3}}...(1&plus;\frac{n} {n})^{\frac{1}{n}}
(بين پرانتز ها ضرب هست)

eh_mn
08-12-2009, 19:30
مقدار حد زير را با تبديل به انتگرال معين بيابيد(n به سمت بي نهايت ميل مي كند)
[ برای مشاهده لینک ، لطفا با نام کاربری خود وارد شوید یا ثبت نام کنید ](1&plus;\frac{1}{n})(1&plus;\frac{2}{n})^{\frac {1}{2}}(1&plus;\frac{3}{n})^{\frac{1}{3}}...(1&plus;\frac{n} {n})^{\frac{1}{n}}
(بين پرانتز ها ضرب هست)

قرار مي‌دهيم
[ برای مشاهده لینک ، لطفا با نام کاربری خود وارد شوید یا ثبت نام کنید ]{i=1}^n(1&plus;\frac{i}{n})^{\frac1 i}
بنابراين
[ برای مشاهده لینک ، لطفا با نام کاربری خود وارد شوید یا ثبت نام کنید ]{i=1}^n(1&plus;\frac{i}{n})^ {\frac1i}=\sum_{i=1}^n\ln(1&plus;\frac{i}{n})^{\frac1i} &space;=\frac{1}n&space;\sum_{i=1}^n\frac{n}i\ln(1&plus;\frac{i}{n} )&space;=&space;\frac{1}n&space;\sum_{i=1}^n\frac{&space;\ln(1&plus;\frac{i}{n} )}{\frac{i}n}
با حد گيري داريم

[ برای مشاهده لینک ، لطفا با نام کاربری خود وارد شوید یا ثبت نام کنید ]{n\to\infty}\ln&space;y_n&space;=&space;\int_0^1\frac {\ln{(1&plus;x)}}{x}\,dx

بعد از محاسبه‌ي انتگرال معين مقدار حد بدست مياد.

bnmnb
08-12-2009, 22:38
قرار مي‌دهيم
[ برای مشاهده لینک ، لطفا با نام کاربری خود وارد شوید یا ثبت نام کنید ]{i=1}^n(1&plus;\frac{i}{n})^{\frac1 i}
بنابراين
[ برای مشاهده لینک ، لطفا با نام کاربری خود وارد شوید یا ثبت نام کنید ]{i=1}^n(1&plus;\frac{i}{n})^ {\frac1i}=\sum_{i=1}^n\ln(1&plus;\frac{i}{n})^{\frac1i} &space;=\frac{1}n&space;\sum_{i=1}^n\frac{n}i\ln(1&plus;\frac{i}{n} )&space;=&space;\frac{1}n&space;\sum_{i=1}^n\frac{&space;\ln(1&plus;\frac{i}{n} )}{\frac{i}n}
با حد گيري داريم

[ برای مشاهده لینک ، لطفا با نام کاربری خود وارد شوید یا ثبت نام کنید ]{n\to\infty}\ln&space;y_n&space;=&space;\int_0^1\frac {\ln{(1&plus;x)}}{x}\,dx

بعد از محاسبه‌ي انتگرال معين مقدار حد بدست مياد.

متشكرم دوست من.
عجب انتگرالي شد اين.ناسره هست جوابش هم كه فكر نكنم .....

mahdishojaee
10-12-2009, 01:26
برای مشاهده محتوا ، لطفا وارد شوید یا ثبت نام کنید
[
jتوی خط سوم چرا y رو فقط در قسمت چپ تساوی ضرب کردین؟
مرسی ولی اشتباهِ.

Hamidkhann
10-12-2009, 01:46
سلام دوستان
دستور یه تابع چند ضابطه ای توی F-TeX چیه؟ دستور WinEdt رو توش آزمایش کردم جواب نداد.
ممنون

mahsa1469
10-12-2009, 12:07
سری فوریه تابع زیر را در فاصله ی داده شده به دست آورید:

[ برای مشاهده لینک ، لطفا با نام کاربری خود وارد شوید یا ثبت نام کنید ](x)=\left\{\begin{matrix} -1 &-2< x<0 \\ 1& 0<x<2 \end{matrix}\right.

singleguy
11-12-2009, 19:26
اگه میشه یه نفر این سوال را برام حال کنه:

معادله صفحه ای که از دو خط زیر به یک فاصله باشه:


[ برای مشاهده لینک ، لطفا با نام کاربری خود وارد شوید یا ثبت نام کنید ]

afshin b
13-12-2009, 20:34
حال کسی هست این انتگرال رو برای من حل کنه؟
ممنون
[ برای مشاهده لینک ، لطفا با نام کاربری خود وارد شوید یا ثبت نام کنید ]{\0&space;}^{\infty&space;}e^{-x^{2}}dx

يه راه حل ديگه براي اين سوال:
حل با استفاده از مختصات قطبي:

[ برای مشاهده لینک ، لطفا با نام کاربری خود وارد شوید یا ثبت نام کنید ]{0}^{\infty&space;}e^{-x^2}dx

و همچنين ميتوان نوشت:
[ برای مشاهده لینک ، لطفا با نام کاربری خود وارد شوید یا ثبت نام کنید ]{0}^{\infty&space;}e^{-y^2}dy

از ضرب دو انتگرال فوق داريم:
[ برای مشاهده لینک ، لطفا با نام کاربری خود وارد شوید یا ثبت نام کنید ]^2=\int_{0}^{\infty&space;}\int_{0}^{\infty&space;} e^{-x^2-y^2}dxdy

حال تغيير متغير قطبي را اعمال ميكنيم[ برای مشاهده لینک ، لطفا با نام کاربری خود وارد شوید یا ثبت نام کنید ]^2=x^2&plus;y^2 و[ برای مشاهده لینک ، لطفا با نام کاربری خود وارد شوید یا ثبت نام کنید ]

[ برای مشاهده لینک ، لطفا با نام کاربری خود وارد شوید یا ثبت نام کنید ]^2=\int_{0}^{\pi&space;/2}\int_{0}^{\infty&space;}e^{-r^2}rdrd\theta
[ برای مشاهده لینک ، لطفا با نام کاربری خود وارد شوید یا ثبت نام کنید ]^2=\int_{0}^{\pi&space;/2}1/2&space;d\theta&space;=\pi&space;/4\Rightarrow&space;I=\sqrt{\pi&space;}/2

davy jones
14-12-2009, 09:30
اگه میشه یه نفر این سوال را برام حال کنه:

معادله صفحه ای که از دو خط زیر به یک فاصله باشه:


[ برای مشاهده لینک ، لطفا با نام کاربری خود وارد شوید یا ثبت نام کنید ]



حل این سوال یه کم وقت گیره. من حوصله ندارم کامل حلشو بنویسم فقط یه راهنمایی میکنم و امیدوارم کمکت کنه.
این که گفته شده صفحه مطلوب باید از هر دو خط به یک فاصله باشه یعنی باید با هر دو خط موازی باشه. بردار هادی خط L برابر است با:
[ برای مشاهده لینک ، لطفا با نام کاربری خود وارد شوید یا ثبت نام کنید ]
و بردار هادی خط H هم برابر است با:
[ برای مشاهده لینک ، لطفا با نام کاربری خود وارد شوید یا ثبت نام کنید ]
و چون صفحه مورد نظر با این دو خط موازیه پس بردار نرمال صفحه بر این دو خط عموده. یعنی بردار نرمال صفحه حاصل ضرب خارجی این دو برداره.
[ برای مشاهده لینک ، لطفا با نام کاربری خود وارد شوید یا ثبت نام کنید ]
حالا فقط کافیه که یک نقطه از صفحه مورد نظر رو بدست بیاریم تا بتونیم معادله صفحه رو بنویسیم. برای این کار معادله پارامتری خطی موازی با بردار نرمال رو در نظر میگیریم و اون رو یکبار با خط L و یکبار با خط H برخورد میدهیم نقطه مورد نظر ما نقطه ایست که وسط دو نقطه بدست آمده قرار دارد.( محاسبه این قسمتش طولانیه و حوصله ندارم) اگه فرض کنیم که اون نقطه [ برای مشاهده لینک ، لطفا با نام کاربری خود وارد شوید یا ثبت نام کنید ] باشه معادله صفحه برابر میشه با:
[ برای مشاهده لینک ، لطفا با نام کاربری خود وارد شوید یا ثبت نام کنید ]

موفق باشین.

alilosting
14-12-2009, 16:03
بچه ها گلاب به روتون ميخواستم برم كارشناسي ارشد شركت كنم ولي با اين سئوالا فهميدم كه من خيلي بوقم

sepehr_x50
14-12-2009, 16:18
سلام.

کسی درباره تقسیم هورنر (هرنر) اطلاعاتی داره؟!

ممنون میشم اگر میدونید روشش رو توضیح بدید.

ممنون

ادیت

پیدا کردم.


[ برای مشاهده لینک ، لطفا با نام کاربری خود وارد شوید یا ثبت نام کنید ]

Aidin.gh73
14-12-2009, 23:08
سلام
یک این معادله رو برام حل کنه:
a+b=3
a^3+b^3=33

davy jones
15-12-2009, 20:58
سلام
یک این معادله رو برام حل کنه:
a+b=3
a^3+b^3=33

a+b=3
a=3-b
a^3=(3-b)^3=33-b^3
27-b^3-27b+9b^2=33-b^3
9b^2-27b-6=0
3b^2-9b-2=0

بقیه اش دیگه واضحه. ریشه های معادله درجه دو رو بدست میاری و توی معادله اول میذاری. ممکنه که هر دو سری از جوابها صحیح باشن و ممکنه هم که یه سری از جوابها درست باشن.

موفق باشین.

Переклад не
17-12-2009, 12:26
با سلام
یه سوال از اعداد مختلط هستش :
z=1-j
Z=-3j
z=2j-4

(زد ها به ترتیب با اندیس یک تا سه هستن)
حالا گفته:
Re(zz-zz
یعنی : ریل زد یک زد دو منهای زد 2 زد یک
آقا حالا من نمیدونم اومده چیکار کرده (در جوابش) این زد ها که بالا نوشتم رو قرینه کرده و تو جواب هم قرینه هارو پس و پیش به هم
ضرب و تفریق کرده(Re(zz-zz

حالا این حلش چیه؟ فقط اون قسمت مزدوج شده و پس و پیش به هم ضرب شدن رو توضیح بدین ملتفت شدم
ممنون

afshin b
17-12-2009, 13:34
با سلام
یه سوال از اعداد مختلط هستش :
z=1-j
Z=-3j
z=2j-4

(زد ها به ترتیب با اندیس یک تا سه هستن)
حالا گفته:
Re(zz-zz
یعنی : ریل زد یک زد دو منهای زد 2 زد یک
آقا حالا من نمیدونم اومده چیکار کرده (در جوابش) این زد ها که بالا نوشتم رو قرینه کرده و تو جواب هم قرینه هارو پس و پیش به هم
ضرب و تفریق کرده(Re(zz-zz

حالا این حلش چیه؟ فقط اون قسمت مزدوج شده و پس و پیش به هم ضرب شدن رو توضیح بدین ملتفت شدم
ممنون

سوالو اشتباه ننوشتي؟!

Переклад не
17-12-2009, 15:20
سوالو اشتباه ننوشتي؟!

والا تا اونجایی که استادمون تو تخته نوشته بود ، فکر کنم صد در صد کپی کردم دیگه
مسئله اینو میخواد که ریل پارانتز باز ، زد1 در زد 2 ، منهای زد 2 در زد 1
این ایمیج و ریلشو باید پیدا کنیم

اینم بگم که بنده اعداد مختلط جدیدا اینارو یاد گرفتم و تقریبا بلد نیستم

afshin b
17-12-2009, 20:37
والا تا اونجایی که استادمون تو تخته نوشته بود ، فکر کنم صد در صد کپی کردم دیگه
مسئله اینو میخواد که ریل پارانتز باز ، زد1 در زد 2 ، منهای زد 2 در زد 1
این ایمیج و ریلشو باید پیدا کنیم

اینم بگم که بنده اعداد مختلط جدیدا اینارو یاد گرفتم و تقریبا بلد نیستم
ما تو اعداد مختلط i شنيديم j نشنيديم!
پس اون زد سه به چه كاري مياد؟!
فرض كنيم سوال درست باشه :
زد يك زد دو= زد دو زد يك
پس zz-zz=0
پس Re(zz-zz)=0

mahsa1469
17-12-2009, 20:58
ما تو اعداد مختلط i شنيديم j نشنيديم!

iبرابر با j است
ما هم توی ریاضی مهندسی j رو جای i قرار می دیم

isma
17-12-2009, 22:43
آیا این سئوال رو کسی دیده ؟ خط اضافی اون کجا باید رسم بشه؟
در مثلث متساوی الساقین ABC که درآن A=80 درجه است نقطه D را درون مثلث طوری در نظر می گیریم که زاویه DCB=30 و زاویه DBC=10 است . زاویه ADB چند درجه است ؟

Переклад не
17-12-2009, 23:49
ما تو اعداد مختلط i شنيديم j نشنيديم!
پس اون زد سه به چه كاري مياد؟!
فرض كنيم سوال درست باشه :
زد يك زد دو= زد دو زد يك
پس zz-zz=0
پس Re(zz-zz)=0


iبرابر با j است
ما هم توی ریاضی مهندسی j رو جای i قرار می دیم

آقا فداتون بشم این رو جوابشو من دارم
اومده گفته برای پیدا کردن ایمیج باید این زد هارو مزدوج کنیم مثلا اومده این کارو کرده تو جواب:
(یک منهای جی)در(سه جی)منهای(منفی سه جی)در(یک به اضافه جی)

چرا باید اینطوری بکنه؟
و در نهایت شیش جی اومده و ریلش صفره

ahmads
18-12-2009, 02:55
سلام دوستان نمیدونم جاش اینجاست یا نه اما به یه سوال بر خوردم که ممنون میشم اگه کسی تو حلش بهم کمک کنه



[ برای مشاهده لینک ، لطفا با نام کاربری خود وارد شوید یا ثبت نام کنید ]

mahsa1469
18-12-2009, 09:06
آقا فداتون بشم این رو جوابشو من دارم
اومده گفته برای پیدا کردن ایمیج باید این زد هارو مزدوج کنیم مثلا اومده این کارو کرده تو جواب:
(یک منهای جی)در(سه جی)منهای(منفی سه جی)در(یک به اضافه جی)

چرا باید اینطوری بکنه؟
و در نهایت شیش جی اومده و ریلش صفره
اینی که شما نوشتید می شه 12 j و Re اون هم درسته 0 می شه..

Переклад не
18-12-2009, 10:01
اینی که شما نوشتید می شه 12 j و Re اون هم درسته 0 می شه..

ممون
حالا میشه بگید چرا اینو اینطوری مزدوج کرده؟
چرا این طوری نوشته؟
(یک منهای جی)در(سه جی)منهای(منفی سه جی)در(یک به اضافه جی)

parsa_n1
18-12-2009, 15:51
اگه کسی از دوستان این قضیه رو از طریق قضیه فشردگی میتونه اثبات کنه ممنون میشم کمکم کنه.


x-sinx~1/6x^3

منظور از 1/6 يك وشش دهم است.
خيلي ضروري هست ممنون ميشم امشب جواب بديد.

isma
18-12-2009, 20:31
به نظر می رسه که اگر 2x-1=y فرض کنی مسئله خیلی ساده تر بشه . اگر باز هم نتونستی حل کنی بگو تا راهنماییت کنم

parsa_n1
18-12-2009, 20:33
كسي نبود مارو ياري كنه ؟
x-sinx~1/6x^3

isma
18-12-2009, 20:51
با توجه به رابطه sinx = x-x^3/3!+x^5/5!-x^7/7!+x^9/9!-... ( بسط ماکلورن sinx )
می توانیم بنویسیم x^3/3!-x^5/5!<x-sinx<x^3/3! که با تقسیم طرفین بر x^3 ( با فرض مثبت بودن x )
و استفاده از قضیه فشردگی براحتی جواب یک ششم به دست می آید . ( حالت منفی بودن x نیز همینطوری هست فقط جهت نامساوی عوض می شود )

mahsa1469
18-12-2009, 21:06
ممون
حالا میشه بگید چرا اینو اینطوری مزدوج کرده؟
چرا این طوری نوشته؟
(یک منهای جی)در(سه جی)منهای(منفی سه جی)در(یک به اضافه جی)
می شه پرانتز ها رو واضح تر بذارید نمی شه تشخیص داد منها کجاست:41:

ahmads
18-12-2009, 21:19
به نظر می رسه که اگر 2x-1=y فرض کنی مسئله خیلی ساده تر بشه . اگر باز هم نتونستی حل کنی بگو تا راهنماییت کنم
قبلا این راه رو امتحان کرده بودم اما بازم جواب رو نگرفتم، به نظرم میرسه که باید یه نکته داشته باشه

parsa_n1
18-12-2009, 21:30
x-sinx~1/6x^3
من اثبات بدون بسط مك لورن وتيلور و همچنين مشتق رو ميخوام.

isma
18-12-2009, 23:54
یه راه دیگه می نویسم :
[ برای مشاهده لینک ، لطفا با نام کاربری خود وارد شوید یا ثبت نام کنید ]

parsa_n1
19-12-2009, 16:41
كسي نيست اين عبارت رو با دانش سوم دبيرستان بدون استفاده از بسط مك لورن - تيلور و مشتق حل كنه؟؟؟؟/

x-sinx~1/6x^3

ahmads
19-12-2009, 18:55
یه راه دیگه می نویسم :

اگه اشتباه نکنم پستتون خالی هست

eh_mn
19-12-2009, 18:56
كسي نيست اين عبارت رو با دانش سوم دبيرستان بدون استفاده از بسط مك لورن - تيلور و مشتق حل كنه؟؟؟؟/

x-sinx~1/6x^3

شايد اينجوري بشه گفت كه:

براي اثبات اين هم ارزي بايد نشون بديم حد حاصل تقسيم طرفين برابر يك ميشه. فرض كنيم اين حد موجود و برابر a باشه يعني

[ برای مشاهده لینک ، لطفا با نام کاربری خود وارد شوید یا ثبت نام کنید ]{x\to&space;0}\frac{x-\sin&space;x}{\frac{1}{6}x^3}=a
با استفاده از از تغيير متغير x=3t داريم

[ برای مشاهده لینک ، لطفا با نام کاربری خود وارد شوید یا ثبت نام کنید ]{x\to&space;0}\frac{x-\sin&space;x}{\frac{1}{6}x^3}=\lim_{t\to&space;0}\frac{3t-\sin&space;3t}{\frac{27}{6}t^3}
اما

[ برای مشاهده لینک ، لطفا با نام کاربری خود وارد شوید یا ثبت نام کنید ]^3&space;t

بنابراين با كمي ساده سازي داريم


[ برای مشاهده لینک ، لطفا با نام کاربری خود وارد شوید یا ثبت نام کنید ]{1}{9}a&space;&plus;&space;\frac{8}{9}

كه نتيجه مطلوب رو ميده!

parsa_n1
20-12-2009, 16:30
شايد اينجوري بشه گفت كه:

براي اثبات اين هم ارزي بايد نشون بديم حد حاصل تقسيم طرفين برابر يك ميشه. فرض كنيم اين حد موجود و برابر a باشه يعني

[ برای مشاهده لینک ، لطفا با نام کاربری خود وارد شوید یا ثبت نام کنید ]
با استفاده از از تغيير متغير x=3t داريم

[ برای مشاهده لینک ، لطفا با نام کاربری خود وارد شوید یا ثبت نام کنید ] _%7Bt%5Cto&space;0%7D%5Cfrac%7B3t-%5Csin&space;3t%7D%7B%5Cfrac%7B27%7D%7B6%7Dt%5E3%7D
اما

[ برای مشاهده لینک ، لطفا با نام کاربری خود وارد شوید یا ثبت نام کنید ]

بنابراين با كمي ساده سازي داريم


[ برای مشاهده لینک ، لطفا با نام کاربری خود وارد شوید یا ثبت نام کنید ] 7D%7B9%7D

كه نتيجه مطلوب رو ميده!

خط دوم رو چجوري ساده كرديد ميشه يك توضيح بديد

isma
20-12-2009, 19:54
قبلا این راه رو امتحان کرده بودم اما بازم جواب رو نگرفتم، به نظرم میرسه که باید یه نکته داشته باشه

[ برای مشاهده لینک ، لطفا با نام کاربری خود وارد شوید یا ثبت نام کنید ]
که بقیه اش دیگه راحت حل میشه

eh_mn
21-12-2009, 09:14
خط دوم رو چجوري ساده كرديد ميشه يك توضيح بديد

با استفاده از تساوي سوم داريم


[ برای مشاهده لینک ، لطفا با نام کاربری خود وارد شوید یا ثبت نام کنید ]{align}\nonumber&space;\frac{3t&space;-&space;\sin&space;3t}{\frac{27}{6}t^3}&space;&&space;=&space;\frac{3t&space;-&space;3\sin&space;t&plus;4\sin^3t}{\frac{27}{6}t^3}&space;\cr&space;&&space;=&space;\frac{1}{9}\left(&space;\frac{t-\sin&space;t}{\frac{1}{6}t^3}&space;\right)&plus;\frac{8}{9}\left(&space; \frac{\sin^3t}{t^3}\right)&space;\end{align}

با حدگيري از طرفين به رابطه‌ي آخر بر حسب a مي‌رسيم

davy jones
21-12-2009, 12:49
با استفاده از تساوي سوم داريم


[ برای مشاهده لینک ، لطفا با نام کاربری خود وارد شوید یا ثبت نام کنید ] B3t&space;-&space;%5Csin&space;3t%7D%7B%5Cfrac%7B27%7D%7B6%7Dt%5E3%7 D&space;&&space;=&space;%5Cfrac%7B3t&space;-&space;3%5Csin&space;t&plus;4%5Csin%5E3t%7D%7B%5Cfrac%7B27%7D%7B6%7 Dt%5E3%7D&space;%5Ccr&space;&&space;=&space;%5Cfrac%7B1%7D%7B9%7D%5Cleft%28&space;%5Cfrac%7B t-%5Csin&space;t%7D%7B%5Cfrac%7B1%7D%7B6%7Dt%5E3%7D&space;%5Crig ht%29&plus;%5Cfrac%7B8%7D%7B9%7D%5Cleft%28&space;%5Cfrac%7B%5 Csin%5E3t%7D%7Bt%5E3%7D%5Cright%29&space;%5Cend%7Balign% 7D

با حدگيري از طرفين به رابطه‌ي آخر بر حسب a مي‌رسيم

شما برای رسیدن به حکم از خود حکم که نمیتونید استفاده کنید. تو پرانتز اول چطوری حد گرفتین؟

parsa_n1
21-12-2009, 18:37
یه راه دیگه می نویسم :
[ برای مشاهده لینک ، لطفا با نام کاربری خود وارد شوید یا ثبت نام کنید ]

خط سوم اون حد چجوري شد 1/9؟؟؟؟؟؟

isma
21-12-2009, 21:53
[ برای مشاهده لینک ، لطفا با نام کاربری خود وارد شوید یا ثبت نام کنید ]

parsa_n1
22-12-2009, 13:53
خط سوم اون ليميت كه شده بود 1/9 رو فقط نوشتيد 3/27
من يه توضيح مي خواستم كه چجوري مي شه 1/9

soolmaz68
22-12-2009, 17:37
سلام. خیلی ممنون که این ناپیک رو راه انداختید
من صرفا بخاطر ریاضی اینجا عضو شدم
جووووووووووووووون من سوالامو جواب بدید
دانشجوی رشته نرم افزارم ترم یک
استاد ریاضیمون واقعا بد درس میده
جیگر همه رو خووووووووووووووون کرده
دستم به دامانت استاد
سوالامو جواب بده با توضیح باشه خیلی ممنون میشم
قربونت برم
فعلا عجله دارم باید برم روضه
میخوام بدونم این تاپیک هنوز هم فعالیت داره یا نه. استاد اگه هنوز هستی به دادما برسی بی زحمت یه پیام واسه من بده
بازم خیلی خیلی ممنون
خدافظ تا بعد

bnmnb
22-12-2009, 19:08
خط سوم اون ليميت كه شده بود 1/9 رو فقط نوشتيد 3/27
من يه توضيح مي خواستم كه چجوري مي شه 1/9
خوب دوست من صورت و مخرج رو بر 3 تقسيم كنيد واضحهكه اين دو كسر با هم برابر هستن

parsa_n1
22-12-2009, 19:15
خوب دوست من صورت و مخرج رو بر 3 تقسيم كنيد واضحهكه اين دو كسر با هم برابر هستن
اين رو كه بچه پيش دبستاني هم مي دونه منظورم اينه كه اون ليميت خط سوم چجوري جوابش 1/9 شد.

soolmaz68
22-12-2009, 21:41
توروخدا من زیاد وقت ندارم یعنی تا اخر چهارشنبه

میشه بگید فرمولای ریاضی رو چطوری باید بذارم تو سایت؟:41:

davy jones
22-12-2009, 21:50
سلام. خیلی ممنون که این ناپیک رو راه انداختید
من صرفا بخاطر ریاضی اینجا عضو شدم
جووووووووووووووون من سوالامو جواب بدید
دانشجوی رشته نرم افزارم ترم یک
استاد ریاضیمون واقعا بد درس میده
جیگر همه رو خووووووووووووووون کرده
دستم به دامانت استاد
سوالامو جواب بده با توضیح باشه خیلی ممنون میشم
قربونت برم
فعلا عجله دارم باید برم روضه
میخوام بدونم این تاپیک هنوز هم فعالیت داره یا نه. استاد اگه هنوز هستی به دادما برسی بی زحمت یه پیام واسه من بده
بازم خیلی خیلی ممنون
خدافظ تا بعد


بستگی داره سوالت چه قدر سخت باشه:27:
در ثانی اینجا بیشتر از یه استاد داریم.:10:
بجای قربون صدقه سوالاتت رو مطرح کن. ولی قبلش نحوه نوشتن فرمولها رو به صورت ریاضی یاد بگیر. برای این کار میتونی به این تاپیک مراجعه کنی:

برای مشاهده محتوا ، لطفا وارد شوید یا ثبت نام کنید

موفق باشین.
88/10/1

isma
22-12-2009, 22:23
اين رو كه بچه پيش دبستاني هم مي دونه منظورم اينه كه اون ليميت خط سوم چجوري جوابش 1/9 شد.

نمی دونم که متوجه منظورت شدم یانه در هر صورت
[ برای مشاهده لینک ، لطفا با نام کاربری خود وارد شوید یا ثبت نام کنید ]

parsa_n1
23-12-2009, 16:05
نمی دونم که متوجه منظورت شدم یانه در هر صورت
[ برای مشاهده لینک ، لطفا با نام کاربری خود وارد شوید یا ثبت نام کنید ]
ممنون خيلي عالي بود

binaeram
23-12-2009, 21:27
با سلام
یه سوال اماری داشتم.اگه 4 بازی شطرنج در حال انجام باشه و در هر بازی 3 احتمال برد باخت و مساوی وجود داشته باشه مجموعا چند حالت می تواند وجود داشته باشد.لطف کنید فرمول مربوطه رو هم بنویسید.

afshin b
23-12-2009, 21:52
با سلام
یه سوال اماری داشتم.اگه 4 بازی شطرنج در حال انجام باشه و در هر بازی 3 احتمال برد باخت و مساوی وجود داشته باشه مجموعا چند حالت می تواند وجود داشته باشد.لطف کنید فرمول مربوطه رو هم بنویسید.
فكر كنم سه به توان چهار حالت ميشه.

binaeram
23-12-2009, 21:55
سپاسگذارم.

singleguy
26-12-2009, 14:59
سلام
سوالی دارم از ریاضی مهندسی
کسی میتونه بدست آوردن معادله ی خم را با استفاده از دو نقطه بهم یاد بده؟
مثلا معادله ی خم زیر چجوری میشه z=ix+x ؟!!!!!

[ برای مشاهده لینک ، لطفا با نام کاربری خود وارد شوید یا ثبت نام کنید ]

afshin b
26-12-2009, 19:28
سلام
سوالی دارم از ریاضی مهندسی
کسی میتونه بدست آوردن معادله ی خم را با استفاده از دو نقطه بهم یاد بده؟
مثلا معادله ی خم زیر چجوری میشه z=ix+x ؟!!!!!


من تاحالا چنين چيزيو نديدم! ولي فكر كنم اينطوري باشه:
ميدونيم كه z=x+yi و از طرفي چون معادله خط نمودارت y=x هست يعني ميشه تو رابطه z جاي x yقرار داد و نوشت z=x+xi.
اگه معادله خطت هر چيز ديگه اي هم باشه ميتوني همونو اعمال كني.

singleguy
26-12-2009, 21:28
من تاحالا چنين چيزيو نديدم! ولي فكر كنم اينطوري باشه:
ميدونيم كه z=x+yi و از طرفي چون معادله خط نمودارت y=x هست يعني ميشه تو رابطه z جاي x yقرار داد و نوشت z=x+xi.
اگه معادله خطت هر چيز ديگه اي هم باشه ميتوني همونو اعمال كني.

نه.... اینجوری نیست!محور توی اعداد مختلط هست توی اعداد حقیقی نیست! اون نمودار y نیست توی شکل اشتباها گذاشته شده.... محور افقی محور حقیقی ها)Re) و محور عمودی محور موهومی ها(Im) هست
مثلا اگه معادله ی منحنی را بخوایم که از حاصل اتصال نقطه ی 1 وi به نقطه ی 2 و i باشه میشه: x+i
(یا به صورت پارامتری t+i)
نمیدونم چجوری شد که اینجوری شد؟

afshin b
27-12-2009, 00:55
نه.... اینجوری نیست!محور توی اعداد مختلط هست توی اعداد حقیقی نیست! اون نمودار y نیست توی شکل اشتباها گذاشته شده.... محور افقی محور حقیقی ها)Re) و محور عمودی محور موهومی ها(Im) هست
مثلا اگه معادله ی منحنی را بخوایم که از حاصل اتصال نقطه ی 1 وi به نقطه ی 2 و i باشه میشه: x+i
(یا به صورت پارامتری t+i)
نمیدونم چجوری شد که اینجوری شد؟
نه عزيز شما داري اشتباه ميكني. براي اعداد مختلط اصلا نمودار نداريم! اين نمودار ماله قسمت حقيقي و قسمت موهومي( همون ضريب i ) هست، يعني ميتوني باهاش مثل يه نمودار اعداد حقيقي برخورد كني.
به احتمال خيلي زياد راه حل من درسته!

davy jones
27-12-2009, 14:46
سلام
سوالی دارم از ریاضی مهندسی
کسی میتونه بدست آوردن معادله ی خم را با استفاده از دو نقطه بهم یاد بده؟
مثلا معادله ی خم زیر چجوری میشه z=ix+x ؟!!!!!

[ برای مشاهده لینک ، لطفا با نام کاربری خود وارد شوید یا ثبت نام کنید ]


همونطور که افشین خان هم گفتند، تو اعداد مختلط نمودار نداریم. ولی اگه فرض کنیم که تو اعداد حقیقی داریم بحث میکنیم، میشه ثابت کرد که از تعداد n نقطه دلخواه که در صفحه xy قرار دارند میتوان یک و فقط یک خم از نوع چند جمله ای با ضرایب حقیقی و از درجه n-1 معین کرد به طوری که از تمام n نقطه مورد نظر عبور کنه. اگه منظورت اینه بگو تا بیشتر کمکت کنم.

موفق باشین.

ramTn
27-12-2009, 16:26
سلام
سوالی دارم از ریاضی مهندسی
کسی میتونه بدست آوردن معادله ی خم را با استفاده از دو نقطه بهم یاد بده؟
مثلا معادله ی خم زیر چجوری میشه z=ix+x ؟!!!!!

[ برای مشاهده لینک ، لطفا با نام کاربری خود وارد شوید یا ثبت نام کنید ]


معادله ي خم پاره خطي كه دو سر اون نقاط A و B هستند:


برای مشاهده محتوا ، لطفا وارد شوید یا ثبت نام کنید

دقت كنيد كه A و B دو عدد مختلط هستند (در اينجا 0 و 1+i ) و t بين 0 و 1 قرار داره.

قاهر - Gahir
29-12-2009, 16:34
سلام دوستان ،

یکی از دوستان زحمت بکشن این سوال پایینی رو برام حل کنند :

اگر n و m اعدادی فرد باشند ، آنگاه ثابت کنید که :

برای مشاهده محتوا ، لطفا وارد شوید یا ثبت نام کنید


لطفا دوستان هر چه سریع تر این رو برام ثابت کنند .


ممنون

davy jones
29-12-2009, 22:05
سلام دوستان ،

یکی از دوستان زحمت بکشن این سوال پایینی رو برام حل کنند :

اگر n و m اعدادی فرد باشند ، آنگاه ثابت کنید که :


برای مشاهده محتوا ، لطفا وارد شوید یا ثبت نام کنید


لطفا دوستان هر چه سریع تر این رو برام ثابت کنند .


ممنون


منظورت از | چیه؟
آیا منظورت بخشپذیریه؟

afshin b
29-12-2009, 23:00
كسي ميتونه اين انتگرالو برام حل كنه؟:
[ برای مشاهده لینک ، لطفا با نام کاربری خود وارد شوید یا ثبت نام کنید ]{dx}{x^3&plus;5x^2&plus;6x}

alihacker2008
29-12-2009, 23:21
بچه ها یکم درباره ی انتگرال گیری به کمک توابع مثلثاتی توضیح میدین

bnmnb
29-12-2009, 23:32
كسي ميتونه اين انتگرالو برام حل كنه؟:
[ برای مشاهده لینک ، لطفا با نام کاربری خود وارد شوید یا ثبت نام کنید ]{dx}{x^3&plus;5x^2&plus;6x}
اول از X فاكتور بگيريد بعد يك عبارت درجه 2 ايجاد مي شه اون رو تفكيك كنيد به(X+3)(X+2) حالا از تفكيك كسر ها استفاده كنيد.:10:

bnmnb
29-12-2009, 23:32
بچه ها یکم درباره ی انتگرال گیری به کمک توابع مثلثاتی توضیح میدین
منظور شما جايگذاري مثلثاتي هست؟

alihacker2008
29-12-2009, 23:38
منظور شما جايگذاري مثلثاتي هست؟

نه منظورم اینه که مثلا یه انتگرال که توش یه رادیکال به صورت ax^2+b وجودداره را با تغییر متغیر مثلثاتی حل کنیم

قاهر - Gahir
30-12-2009, 16:09
منظورت از | چیه؟
آیا منظورت بخشپذیریه؟
سلام . آره دوست عزیز ، منظورم همون بود ( رابطه‌ی عاد کردن !)

ولی جوابش آسون بود ، خودم پیداش کردم : البته ، سوالم رو ویرایش کردم چون بین m² و n² علامت منفی بود ! (چون اگه مثبت باشه ، اونوقت عاد نمیکنه)

حالا جواب مسئله :

چون n و m اعدادی فرد هستند پس مربعشون به صورت 8q + 1 نوشته میشه یعنی :


برای مشاهده محتوا ، لطفا وارد شوید یا ثبت نام کنید
*چون حاصل ضرب هر دو عدد متوالی ، عددی است زوج ، پس (k(k+1 عددی زوج هست (2q) .
این عمل برای n که اون هم فرد هست ، صادقه .

حالا :



برای مشاهده محتوا ، لطفا وارد شوید یا ثبت نام کنید

یعنی ثابت شد .

emgjey
31-12-2009, 19:25
با سلام
دایره ای با شعاع R داریم وتر AB آن را در نقطه ی معلومی قطع میکند
مرکز هندسی قطعه ایجاد شده نسبت به یک محور انتخابی چجوری به دست میاد؟

davy jones
02-01-2010, 11:08
با سلام
دایره ای با شعاع R داریم وتر AB آن را در نقطه ی معلومی قطع میکند
مرکز هندسی قطعه ایجاد شده نسبت به یک محور انتخابی چجوری به دست میاد؟

کدوم یکی از قطعات؟ دو تا قطعه داریم. با شکل توضیح بده.

emgjey
02-01-2010, 11:48
کدوم یکی از قطعات؟ دو تا قطعه داریم. با شکل توضیح بده.
منظور قطعه ی کوچکتر هست
در واقع مقدار X رو در این شکل میخوام:
[ برای مشاهده لینک ، لطفا با نام کاربری خود وارد شوید یا ثبت نام کنید ]

davy jones
02-01-2010, 13:00
منظور قطعه ی کوچکتر هست
در واقع مقدار X رو در این شکل میخوام:
[ برای مشاهده لینک ، لطفا با نام کاربری خود وارد شوید یا ثبت نام کنید ]

در واقع باید مرکز ثقل قطاع کوچکتر رو پیدا کنیم:
فرض میکنیم که چگالی توزیع جرم روی کل دایره بصورت f(x,y) باشه که البته در اینجا f=1
برای پیدا کردن مرکز جرم باید دو تا انتگرال 2 گانه بگیریم که اولی x نقطه مرکز ثقل و دومی y اون نقطه رو معلوم میکنه که از همین الان واضحه که y نقطه مورد نظر به دلیل تقارن شکل صفره. (ضمنا فرض میکنیم که کمترین فاصله وتر AB تا مرکز دایره برابر با r باشه r=R.cos a):

[ برای مشاهده لینک ، لطفا با نام کاربری خود وارد شوید یا ثبت نام کنید ] E%7BR%7D&space;%5Cint_%7B-%5Csqrt%7BR%5E%7B2%7D-x%5E%7B2%7D%7D%7D%5E%7B&plus;%5Csqrt%7BR%5E%7B2%7D-x%5E%7B2%7D%7D%7D&space;xf%28x,y%29dydx%7D%7B%5Cint_%7Br %7D%5E%7BR%7D%5Cint_%7B-%5Csqrt%7BR%5E%7B2%7D-x%5E%7B2%7D%7D%7D%5E%7B&plus;%5Csqrt%7BR%5E%7B2%7D-x%5E%7B2%7D%7D%7D&space;f%28x,y%29dydx%7D&space;,f%28x,y%29=1% 5CRightarrow&space;%5Cbar%7Bx%7D=%5Cfrac%7B%5Cint_%7Br%7 D%5E%7BR%7D&space;2x%5Csqrt%7BR%5E%7B2%7D-x%5E%7B2%7D%7D&space;dx%7D%7B%5Cint_%7Br%7D%5E%7BR%7D&space;2% 5Csqrt%7BR%5E%7B2%7D-x%5E%7B2%7D%7D&space;dx%7D=%5Cfrac%7B%5Cfrac%7B-2%7D%7B3%7D%28R%5E%7B2%7D-r%5E%7B2%7D%29%5E%7B%5Cfrac%7B3%7D%7B2%7D%7D%7D%7B I%7D

که در اینجا I انتگرال مخرج هستش که حوصله حساب کردنشو ندارم.

موفق باشین.

ramTn
04-01-2010, 00:03
منظور قطعه ی کوچکتر هست
در واقع مقدار X رو در این شکل میخوام:


با مختصات قطبي مي شه انتگرال رو محاسبه كرد:





مساحت S هم بدون استفاده از انتگرال قابل محاسبه است (ابتدا مساحت قطاع را محاسبه كنيد و سپس مساحت مثلث را از آن كم كنيد) :



[ برای مشاهده لینک ، لطفا با نام کاربری خود وارد شوید یا ثبت نام کنید ] Rightarrow&space;%5Cvec%7Bm%7D=&space;%5Cfrac%7B2%7D%7B3%7DR%5 Cfrac%7Bsin%5E3&space;%5Calpha&space;%7D%7B%5Calpha&space;-%5Cfrac%7B1%7D%7B2%7Dsin2%5Calpha&space;%7D&space;%5Chat%7Bx%7 D

singleguy
04-01-2010, 22:35
سلام
من در یه درس دیگه در بخشی که مربوط به ریاضیات میشد دچار مشکل شدم! اگه میشه یه نفر بگه در عبارت زیر، دو قسمتی که با رنگ قرمز مشخص کردم، چرا با هم برابر هستند؟!

[ برای مشاهده لینک ، لطفا با نام کاربری خود وارد شوید یا ثبت نام کنید ]

davy jones
05-01-2010, 01:22
سلام
من در یه درس دیگه در بخشی که مربوط به ریاضیات میشد دچار مشکل شدم! اگه میشه یه نفر بگه در عبارت زیر، دو قسمتی که با رنگ قرمز مشخص کردم، چرا با هم برابر هستند؟!

[ برای مشاهده لینک ، لطفا با نام کاربری خود وارد شوید یا ثبت نام کنید ]


باید هر کدوم از عبارتهای قرمز سمت چپ رو به صورت سینوی و کسینوسی بنویسی و قسمت ریل و ایمجینری رو از هم جدا کنی. یعنی به زبون ساده تر:
2cos72-3cos35-8cos115=3.94cos66.98
البته باید دقت کرد که اعداد داخل کسینوس بر حسب درجه هستند نه رادیان.

davy jones
05-01-2010, 01:24
سلام
من در یه درس دیگه در بخشی که مربوط به ریاضیات میشد دچار مشکل شدم! اگه میشه یه نفر بگه در عبارت زیر، دو قسمتی که با رنگ قرمز مشخص کردم، چرا با هم برابر هستند؟!

[ برای مشاهده لینک ، لطفا با نام کاربری خود وارد شوید یا ثبت نام کنید ]


باید هر کدوم از عبارتهای قرمز سمت چپ رو به صورت سینوی و کسینوسی بنویسی و قسمت ریل و ایمجینری رو از هم جدا کنی. یعنی به زبون ساده تر:
2cos72-3cos35-8cos115=3.94cos66.98
البته باید دقت کرد که اعداد داخل کسینوس بر حسب درجه هستند نه رادیان.

singleguy
05-01-2010, 13:19
باید هر کدوم از عبارتهای قرمز سمت چپ رو به صورت سینوی و کسینوسی بنویسی و قسمت ریل و ایمجینری رو از هم جدا کنی. یعنی به زبون ساده تر:
2cos72-3cos35-8cos115=3.94cos66.98
البته باید دقت کرد که اعداد داخل کسینوس بر حسب درجه هستند نه رادیان.
خیــــــــــــــــــلی ممنون!:11:
ولی ببخشید من اینقدر خنگم :دی خب حالا اون عبارتی که خوتون گفتید علت تساویش چیه؟! :دی چطوری این سه تا کسینوس تبدیل شد به یکی؟! ضریبش و عبارت داخل کسینوس چطوری بدست اومدن؟!

ramTn
05-01-2010, 15:03
خیــــــــــــــــــلی ممنون!:11:
ولی ببخشید من اینقدر خنگم :دی خب حالا اون عبارتی که خوتون گفتید علت تساویش چیه؟! :دی چطوری این سه تا کسینوس تبدیل شد به یکی؟! ضریبش و عبارت داخل کسینوس چطوری بدست اومدن؟!

سلام،
بینید دوست من، هر عدد مختلط یک نمایش دکارتی داره که در این نمایش طول و عرض اون عدد مختلط ذکر میشه. نمایش دکارتی به صورت [ برای مشاهده لینک ، لطفا با نام کاربری خود وارد شوید یا ثبت نام کنید ] یا به صورت [ برای مشاهده لینک ، لطفا با نام کاربری خود وارد شوید یا ثبت نام کنید ] هستش. وقتی دو عدد مختلط جمع می شوند، کافیه مثل دو تا بردار ساده طول و عرض هر کدوم رو جمع کنیم : [ برای مشاهده لینک ، لطفا با نام کاربری خود وارد شوید یا ثبت نام کنید ]

ولی در نمایش قطبی یک عدد مختلط اندازه ی بردار اون و زاویه ای که اون بردار با محور حقیقی می سازه ذکر می شه، که به صورت [ برای مشاهده لینک ، لطفا با نام کاربری خود وارد شوید یا ثبت نام کنید ] یا [ برای مشاهده لینک ، لطفا با نام کاربری خود وارد شوید یا ثبت نام کنید ] نمایش داده می شه.

برای تبدیل این دو به هم اتحادهای زیر یادت باشه:

[ برای مشاهده لینک ، لطفا با نام کاربری خود وارد شوید یا ثبت نام کنید ] Ctheta&space;&plus;isin%5Ctheta&space;%5Cright&space;%29

[ برای مشاهده لینک ، لطفا با نام کاربری خود وارد شوید یا ثبت نام کنید ]

[ برای مشاهده لینک ، لطفا با نام کاربری خود وارد شوید یا ثبت نام کنید ]

singleguy
05-01-2010, 21:22
ممنون از راهنمایی آقای ramtn ولی من سوالم این بود که آقای davy jones چطوری اون سه تا کسینوس را تبدیل کردن به یک کسینوس
اصلا اگه میشه روند تبدیل2cos72-3cos35-8cos115 به عبارت 3.94cos66.98 را برام نمایش بدید
واقعا شرمندم :دی چیکار کنم دیگه مغزم کشش نداره برای این چیزا :دی
پیشاپیش تشکرمیکنم! :11:

davy jones
05-01-2010, 21:36
ممنون از راهنمایی آقای ramtn ولی من سوالم این بود که آقای davy jones چطوری اون سه تا کسینوس را تبدیل کردن به یک کسینوس
اصلا اگه میشه روند تبدیل2cos72-3cos35-8cos115 به عبارت 3.94cos66.98 را برام نمایش بدید
واقعا شرمندم :دی چیکار کنم دیگه مغزم کشش نداره برای این چیزا :دی
پیشاپیش تشکرمیکنم! :11:

خب خودت یه ماشین حساب بردار و کسینوس 72 درجه رو در 2 ضرب کن بعد با منفی حاصلضرب 3 در کسینوس 35 جمعش کن و در نهایت با منفی حاصلضرب 8 در کسینوس 115 درجه جمعش کن و ببین که آیا با حاصلضرب 3.94 در کسینوس 66.98 برابره یا نه.
این که یهو سه تا کسینوس شده یه کسینوس، چیز خارق العاده ای نیست (یعنی مثلا اتحاد مثلثاتی خاصی نداره) اگه واقعا میخوای بفهمی که چطوری اینطوری میشه، دوباره به پست جناب ramTn مراجعه کنید. سوال شما این بود که چطوری دو طرف معادله با هم مساویند و من هم عبارت رو برای شما ساده کردم تا شما براحتی درستی اون رو تحقیق کنی.

موفق باشین.
88/10/15

mr.reza
06-01-2010, 16:52
دوستان کسی میتونه این و اثبات کنه !


قضيه :

Limf (x)
x->a


وجود دارد اگر تنها اگر

Lim f (x)
x--->a+


Lim f (x) ,
x--->a-
وجود داشته و با هم برابر باشند.

ramTn
06-01-2010, 23:29
دوستان کسی میتونه این و اثبات کنه !


قضيه :

Limf (x)
x->a


وجود دارد اگر تنها اگر

Lim f (x)
x--->a+


Lim f (x) ,
x--->a-
وجود داشته و با هم برابر باشند.

كافيه تعريف اين سه گزاره اي كه شما نوشتيد رو مرور كنيم:

[ برای مشاهده لینک ، لطفا با نام کاربری خود وارد شوید یا ثبت نام کنید ] 8&space;x&space;%5Cright&space;%29=A يعني به ازاي هر [ برای مشاهده لینک ، لطفا با نام کاربری خود وارد شوید یا ثبت نام کنید ] يك [ برای مشاهده لینک ، لطفا با نام کاربری خود وارد شوید یا ثبت نام کنید ] وجود دارد كه اگر [ برای مشاهده لینک ، لطفا با نام کاربری خود وارد شوید یا ثبت نام کنید ] آنگاه داشته باشيم [ برای مشاهده لینک ، لطفا با نام کاربری خود وارد شوید یا ثبت نام کنید ]

[ برای مشاهده لینک ، لطفا با نام کاربری خود وارد شوید یا ثبت نام کنید ] %28x%29=A%5E%7B&plus;%7D [ برای مشاهده لینک ، لطفا با نام کاربری خود وارد شوید یا ثبت نام کنید ] %5Cleft&space;%28&space;x&space;%5Cright&space;%29=A%5E%7B&plus;%7D يعني به ازاي هر [ برای مشاهده لینک ، لطفا با نام کاربری خود وارد شوید یا ثبت نام کنید ] يك [ برای مشاهده لینک ، لطفا با نام کاربری خود وارد شوید یا ثبت نام کنید ] وجود دارد كه اگر [ برای مشاهده لینک ، لطفا با نام کاربری خود وارد شوید یا ثبت نام کنید ] آنگاه [ برای مشاهده لینک ، لطفا با نام کاربری خود وارد شوید یا ثبت نام کنید ]

[ برای مشاهده لینک ، لطفا با نام کاربری خود وارد شوید یا ثبت نام کنید ] يعني به ازاي هر [ برای مشاهده لینک ، لطفا با نام کاربری خود وارد شوید یا ثبت نام کنید ] يك [ برای مشاهده لینک ، لطفا با نام کاربری خود وارد شوید یا ثبت نام کنید ] وجود دارد كه اگر [ برای مشاهده لینک ، لطفا با نام کاربری خود وارد شوید یا ثبت نام کنید ] آنگاه [ برای مشاهده لینک ، لطفا با نام کاربری خود وارد شوید یا ثبت نام کنید ]



از گزاره ي اول به راحتي ميشه گزاره هاي بعدي رو نتيجه گرفت، چون اگر [ برای مشاهده لینک ، لطفا با نام کاربری خود وارد شوید یا ثبت نام کنید ] در شرايط گزاره ي اول صدق كند، [ برای مشاهده لینک ، لطفا با نام کاربری خود وارد شوید یا ثبت نام کنید ] در شرايط گزاره ي دوم و سوم صدق مي كنند.

از دو گزاره ي دوم و سوم هم مي شه گزاره ي اول رو نتيجه گرفت، فرض كنيد [ برای مشاهده لینک ، لطفا با نام کاربری خود وارد شوید یا ثبت نام کنید ] داده شده، در اين صورت بنا به فرض به ازاي [ برای مشاهده لینک ، لطفا با نام کاربری خود وارد شوید یا ثبت نام کنید ] يك [ برای مشاهده لینک ، لطفا با نام کاربری خود وارد شوید یا ثبت نام کنید ] و همچنين به ازاي [ برای مشاهده لینک ، لطفا با نام کاربری خود وارد شوید یا ثبت نام کنید ] يك [ برای مشاهده لینک ، لطفا با نام کاربری خود وارد شوید یا ثبت نام کنید ] وجود دارد كه به ترتيب شرايط گزاره هاي دوم و سوم را ارضا كنند. كافي است قرار دهيد [ برای مشاهده لینک ، لطفا با نام کاربری خود وارد شوید یا ثبت نام کنید ] 1%7D,%5Cdelta&space;_%7B2%7D&space;%5Cright&space;%5C%7D تا به وسيله ي [ برای مشاهده لینک ، لطفا با نام کاربری خود وارد شوید یا ثبت نام کنید ] شرايط گزاره ي اول هم برقرار شود.

iranch
08-01-2010, 17:28
کمک فوری
دوستان امتحان دارم لطفا کمکم کنید
4 تا سوال دارم
در مورد انتگراله
خواستم فرمول ها بنویسم دیدم نمیشه تو سیستم
مجبور شدم عکس بگیرم
لطفا کمک کنید کف کردم بستگی فکر کردم و نتیجه نداد :41:

این عکس اول

برای مشاهده محتوا ، لطفا وارد شوید یا ثبت نام کنید
به توان 9 رسیده
اعدادش هم 2 هست
نمیدانم چطوری du رو بدست اورده :41: اصلا این du من نمیتونم بگیرم چرا




عکس3

برای مشاهده محتوا ، لطفا وارد شوید یا ثبت نام کنید
این du چطوری گرفته؟ 3 رو از کجا اضافه کرد؟ چرا 3 گذاشت؟

عکس 4

برای مشاهده محتوا ، لطفا وارد شوید یا ثبت نام کنید

اول کسر انتگرال هست u و du چطوری گرفت؟ 3 و 2 از کجا امدند؟
اعداد هم 3 هست فقط اون عدد پشت رادیکال du تو مخرج 2 هست


عکس دوم

برای مشاهده محتوا ، لطفا وارد شوید یا ثبت نام کنید
این انتگرال هستش f(b-f(a
انتگرال 2 و 0 هست اون رادیکال که روش خط کشیدم بعد نتیجه داده 9*3 من گیر کردم که رادیکال چطوری شده 9 ضرب در 3 این از 2 (انتگرال 2 و 0 هست) 2 تمام شد حالا جای x که صفر میزاریم میشه 4تقسیم بر 9 درست هم هست
اما وقتی پرانتز میشه 0 خوب کلش ضربدر 0 بشه میشه صفر اما اینجا 4تقسیم بر 9 جان سالم به در برده چرا؟

(عکس ها با ie یا firefox باز شه فکر کنم بهتره)
اقا کمک

iranch
08-01-2010, 18:19
بچه ها من رو یادتون نره
منتظرم شرمنده

davy jones
08-01-2010, 19:06
کمک فوری
دوستان امتحان دارم لطفا کمکم کنید
4 تا سوال دارم
در مورد انتگراله
خواستم فرمول ها بنویسم دیدم نمیشه تو سیستم
مجبور شدم عکس بگیرم
لطفا کمک کنید کف کردم بستگی فکر کردم و نتیجه نداد :41:

این عکس اول

برای مشاهده محتوا ، لطفا وارد شوید یا ثبت نام کنیدبه توان 9 رسیده
اعدادش هم 2 هست
نمیدانم چطوری du رو بدست اورده :41: اصلا این du من نمیتونم بگیرم چرا




عکس3

برای مشاهده محتوا ، لطفا وارد شوید یا ثبت نام کنیداین du چطوری گرفته؟ 3 رو از کجا اضافه کرد؟ چرا 3 گذاشت؟

عکس 4

برای مشاهده محتوا ، لطفا وارد شوید یا ثبت نام کنیداول کسر انتگرال هست u و du چطوری گرفت؟ 3 و 2 از کجا امدند؟
اعداد هم 3 هست فقط اون عدد پشت رادیکال du تو مخرج 2 هست


عکس دوم

برای مشاهده محتوا ، لطفا وارد شوید یا ثبت نام کنیداین انتگرال هستش f(b-f(a
انتگرال 2 و 0 هست اون رادیکال که روش خط کشیدم بعد نتیجه داده 9*3 من گیر کردم که رادیکال چطوری شده 9 ضرب در 3 این از 2 (انتگرال 2 و 0 هست) 2 تمام شد حالا جای x که صفر میزاریم میشه 4تقسیم بر 9 درست هم هست
اما وقتی پرانتز میشه 0 خوب کلش ضربدر 0 بشه میشه صفر اما اینجا 4تقسیم بر 9 جان سالم به در برده چرا؟

(عکس ها با ie یا firefox باز شه فکر کنم بهتره)
اقا کمک

در مورد عکس اول:

u=1-2x
du/dx=-2
du=-2dx
dx=-1/2 du

عکس دوم:

یه بار بجای x بزار 2. دو به توان 3 میشه 8. 8 به علاوه یک میشه 9. بعد به توان سه دوم میرسه که توان صورت مستقیم اعمال شده و توان مخرج به صورت فرجه ظاهر میشه. اگه بجای x بزاری صفر، از توی پرانتز فقط یک میمونه که به هر توانی برسونیش، بازم جواب یک میشه.

عکس سوم:

[ برای مشاهده لینک ، لطفا با نام کاربری خود وارد شوید یا ثبت نام کنید ]

عکس چهارم:

u یک متغیر کمکی هستش که به صورت رادیکال 3x-1 تعریف شده تا انتگرال گیری راحتتر بشه. چون متغیر x رو به u تغییر داده حتما باید dx رو هم بر حسب du نوشته بشه. (تو عکسهای قبلی هم u متغیر کمکیه.)

در کل فکر میکنم که شما چیزی از انتگرال نمیدونین.


موفق باشین.

iranch
08-01-2010, 20:15
تشکر از راهنمایی شما
راستش همش با 10 ریاضی رو پاس کردیم الان گیر کردیم باید همه چیز رو بخوانیم آن هم چند برابر
شرمنده هستم برم روی پاسخ های شما تاملی کنم

iranch
08-01-2010, 20:54
در مورد عکس اول:

u=1-2x
du/dx=-2
du=-2dx
dx=-1/2 du

عکس دوم:

یه بار بجای x بزار 2. دو به توان 3 میشه 8. 8 به علاوه یک میشه 9. بعد به توان سه دوم میرسه که توان صورت مستقیم اعمال شده و توان مخرج به صورت فرجه ظاهر میشه. اگه بجای x بزاری صفر، از توی پرانتز فقط یک میمونه که به هر توانی برسونیش، بازم جواب یک میشه.

عکس سوم:

[ برای مشاهده لینک ، لطفا با نام کاربری خود وارد شوید یا ثبت نام کنید ]

عکس چهارم:

u یک متغیر کمکی هستش که به صورت رادیکال 3x-1 تعریف شده تا انتگرال گیری راحتتر بشه. چون متغیر x رو به u تغییر داده حتما باید dx رو هم بر حسب du نوشته بشه. (تو عکسهای قبلی هم u متغیر کمکیه.)

در کل فکر میکنم که شما چیزی از انتگرال نمیدونین.


موفق باشین.
عزیز با عرض شرمندگی

این فرمول du/dx رو من بلد نبودم خیلی کمک کرد (عالی بود)


عکس دوم بردنش تو رادیکال مشکل ندارم (مشکل دومی از این عکس با توضیح شما حل شد)
انتگرال 2 و 0 هست اون رادیکال که روش خط کشیدم بعد نتیجه داده 9*3 من گیر کردم که رادیکال چطوری شده 9 ضرب در 3


عکس سوم هم تا اخرش فهمیدم فقط 3تقسیم بر 2 از کجا امد؟ چون u و du رو جایگزین کردیم ولی این 3/2 رو نمیدونم از کجا امد

عکس چهارم
مشکل تو du هست
اصلا نمیدانم از کجا امد
2 تا x هم داریم (du/dx نمیدانم کدام از x ها رو برابر du/dx قرار بدهم)
تو du اصلا نمیدانم 3 صورذت از کجا امد؟ یا 2 پشت رادیکال در مخرج
فکر کنم فرمول باشد که نمیدانم


ببخشید

davy jones
08-01-2010, 22:18
انتگرال 2 و 0 هست اون رادیکال که روش خط کشیدم بعد نتیجه داده 9*3 من گیر کردم که رادیکال چطوری شده 9 ضرب در 3 9 به توان 3 رسیده. ضرب نشده.


عکس سوم هم تا اخرش فهمیدم فقط 3تقسیم بر 2 از کجا امد؟ چون u و du رو جایگزین کردیم ولی این 3/2 رو نمیدونم از کجا امد
du=3x^2dx
در نتیجه: 2x^2dx=3/2du


عکس چهارم
مشکل تو du هست
اصلا نمیدانم از کجا امد
2 تا x هم داریم (du/dx نمیدانم کدام از x ها رو برابر du/dx قرار بدهم)
تو du اصلا نمیدانم 3 صورذت از کجا امد؟ یا 2 پشت رادیکال در مخرج
فکر کنم فرمول باشد که نمیدانم
شما اصلا بلدی مشتق بگیری؟


این فرمول du/dx رو من بلد نبودم خیلی کمک کرد (عالی بود)

الان مطمئن شدم که بلد نیستس مشتق بگیری.

iranch
08-01-2010, 22:48
[QUOTE=davy jones;4562739]9 به توان 3 رسیده. ضرب نشده.

du=3x^2dx
در نتیجه: 2x^2dx=3/2du

شما اصلا بلدی مشتق بگیری؟



الان مطمئن شدم که بلد نیستس مشتق بگیری.

نوکرتم


du=3x^2dx
در نتیجه: 2x^2dx=3/2du

قاطی پاتی شده :41:



مشتق رو شک بهش کردم ولی اخه چرا باید این کسر رو ازش مشتق بگیریم تو وسط انتگرال؟


الان مطمئن شدم که بلد نیستس مشتق بگیری
این فرمول که ربطی به مشتق نداره
مگر du/dx مال مشتقه؟ اصلا اشاره ای هم نشده تو جزوه بهش:41:



به توان 3 رسیده. ضرب نشده.
نه دادش جواب اخر این شده
4/9 * 9*3
منهای 4/9
برابر با 104/9

davy jones
09-01-2010, 09:13
نقل قول:
du=3x^2dx
در نتیجه: 2x^2dx=3/2du
قاطی پاتی شده [ برای مشاهده لینک ، لطفا با نام کاربری خود وارد شوید یا ثبت نام کنید ]



ببین به زبون خیلی ساده اگر اسم تابع ایکس به توان 3 رو u بزاریم (یعنی از تابع کمکی u که خودمون تعریف کردیم استفاده کنیم) اونوقت جزء دیفرانسیل u صبق تعریف مشتق میشه:
du=3x^2dx
ولی ما در انتگرالی که گذاشتی این عبارت رو داریم:
2x^2dx
پس در نتیجه این عبارت بر حسب du برابر است با :
3/2du
الان دوزاری ت افتاد؟


اصلا اشاره ای هم نشده تو جزوه بهش[ برای مشاهده لینک ، لطفا با نام کاربری خود وارد شوید یا ثبت نام کنید ]


واقعا که ::13::13::13::13::13::13::13::13:


نقل قول:
به توان 3 رسیده. ضرب نشده.
نه دادش جواب اخر این شده
4/9 * 9*3
منهای 4/9
برابر با 104/9

به نظر شما 9 به توان سه دوم میشه چند؟ میشه 27. حالا دوباره به نظر شما 3*9 میشه چند؟ بازم دوباره میشه 27. :31:
دوست عزیز! شما واقعا اینقدر تو ریاضی ضعیفی یا اینکه ما رو گرفتی؟:27:


در ضمن موقع عکس گرفتن با موبایل، فوکوس کن. پدرم در اومد تا تونستم بفهمم چی تو عکسا بود. در ضمن لینک عکسات هم دیگه الن پاک شده.

موفق باشین.
88/10/19

davy jones
10-01-2010, 13:06
سلام. یه سوال جالب برام پیش اومد که خواهش میکنم اساتید پاسخ بدن:
ثابت کنید که هر عدد زوجی را میتوان به صورت مجموع دو عدد اول نوشت. مانند: 20=17+3

amir_rahmani
10-01-2010, 13:12
سلام. یه سوال جالب برام پیش اومد که خواهش میکنم اساتید پاسخ بدن:
ثابت کنید که هر عدد زوجی را میتوان به صورت مجموع دو عدد اول نوشت. مانند: 20=17+3

این غلطه زیرا مثل نقض هم داره
2+3=5